PDA

Просмотр полной версии : Гипотеза ретрансляции масс


<KANE>
15.06.2010, 16:39
Как вы думаете, есть ли в реальной жизни место Масс Ретрансляторам, упомянутым в одноимённой научно-фантастической игре, "Mass Effect" а так же в книгах посвещенных этой вселенной? Может ли существовать технология, позволяющая перемещаться по галактике, намного превышая скорость света?



Просьба не флудить.

Dragon27
15.06.2010, 17:38
Почитай лучше:
http://arxiv.org/abs/gr-qc/0603060
http://en.wikipedia.org/wiki/Faster-than-light

iLLi-DAN
21.06.2010, 19:35
Этот вопрос я себе задал еще со времен выхода первого масс ефекта, и времени на размышление и исследования у меня было предостаточно. Я вообще большой почитатель научной фантастики, именно научной меня всегда интересовали пути развития технологий в будущем, не секрет что большинство технологий придуманных фантастами уже так или иначе воплотились в жизнь. Как мне кажется, любое фантастическое изобретение начинается с мечтаний, я не знаю откуда разрабы масс ефекта эту идею о генераторах массы, но как человек которого черные дыры интересуют с третьего класса скажу что вселенная игры и ее технологии продуманы колоссально.

Все технологии масса не противоречат современной физике хотя бы даже преодоление скорости света. Как известно из теории относительности тело не может достичь скорости света потому что его масса становится бесконечно большой при приближении к С но у нас то есть генератор масс поля который снижает массу корабля до бесконечно маленькой(корабль становится пенопластовым) что позволяет нам развивать скороссти в разы превышающие световую. Отдельно хочется сказать про ретрансляторы это единственная идея мгновенного перемещения где объект телепортации не уничтожается. Большинство фантастических телепортов работают по принципу превращения материи в энергию и переда ее с последующим сбором в точке назначения по сути убийство оригинала и воссоздание копии (клонирование) может кто видел серию биг бенг где шелдон рассказывает что он никогда не будет телепортироватся забавно но на этом моменте я долго смеялся так как за пол года до просмотра я рассказывал своему другу то же самое, что я никогда не буду телепортироватся потому что оригинальный я будет уничтожен. А вот в ретранслятор я бы залетел, так как действует совсем другой принцип и что бы понять как именно нужно уяснить что такое масса. Тут можно начинать проводить параллели с реальностью – масса по современному определению это физическая мера вещества, но почему больше вещества труднее сдвинуть с места, а меньше легче выход МЕ как раз совпал со строительством коллайдера, а в нем сейчас питаются найти частицу отвечающую за массу вещества бозон хигса, частицу которая создает поле наделяющее массой всю материю, то есть массы как таковой нет, есть сопротивление вещества глобальному полю (иной немеханической формы существования материи – вакуум не есть пустота) это сопротивление похоже на сопротивление шарика погруженного в воду, масса шарика не увеличилась, но предавать ему ускорение стало значительно труднее по сравнению с воздухом. Размер шарика также влияет на сопротивляемость ускорению. Прямым доказательством того что вакуум не пустота есть световые волны (колеблется же что-то) свет это не частица это колебания среды подобно звуку. Наши физики чтоб объяснить хотя бы давление света придумали фотон (которого нет) и никто не может сказать то ли свет волна то ли частица. Звук имеет почти такие же свойства как и свет, это колебания воздуха которое имеет давление и от движения источника звука не зависит его скорость а только частота, и чем мощнее источник тем выше частота со светом то же самое, рентгеновские лучи имеют скорость света но частоту и соответственно энергию на много больше. То, что три отсутствии поля гравитации люди не имеют веса и летают никого не удивляет, а в безвоздушном пространстве молоток и пушинка падают с одинаковой скоростью. Если же полностью изолировать тело от источника поля массы оно будет перемещается под воздействием импульса мгновенно что и наблюдается в ретрансляторах, между двумя ретрансляторами создается коридор в который не проникает глобальное поле массы, видели как властелин вышел из ретранслятора мгновенно приобретая нормальную скорость, так же как и нормандия в первом ролике второй части отключив масс генератор сразу перешла со сверхсветовой на нормальную скорость у корабля фактически НЕТ ИНЕРЦИИ (с этим эффектом связана ошибка в работе оружия). Допустим человечество все таки смогло создать корабль который бы разгонялся до сверхсветовой КАК ТОРМОЗИТЬ? В МЕ эта проблема отсутствует. Насчет оружия, в МЕ обычные гауски снабжаются маленьким масс генератором который на участке ствола создает поле с пониженной массой что позволяет разогнать снаряд до бешенных скоростей но со снарядом вылетевшим из масс поля произойдет то же самое что и с нормандией при отключении масс поля он резко затормозится, такая вот фишка – недоглядели. Масс ген никак бы не повлиял на гауски сколько сообщили энергии снаряду столько и осталось. Через 150 лет если создадут компактный ядерный реактор или чет другое в таком оружии итак будет нереальная мощь.
Теперь когда ясно как работает большинство технологий МЕ и понятно что это не просто выдумка можно ответить на вопрос возможны ли такие технологии в будущем.
Они возможны, но не с современной физикой, которая запуталась в дебрях математики и уже совершенно не понимает, что она описывает своими формулами. Для начала нужно понять, что за явление мы описываем и как оно происходит, а потом уже описывать это формулам, формулы были придуманы для облегчения понимания процессов и вычислений – ПОЛУЧИЛОСЬ НАОБОРОТ. Мне всегда было важно понимать суть вещей, суть физических процессов как они происходят? И МЕ подтолкнул меня на исследования природы массы и теории относительности. Раньше В Т.О я просто верил, видел что есть нестыковки но верил, один физик мне сказал что ее понимают три человека на земле и один из них уже умер. То, что я обнаружил в теории относительности меня просто поразило, чего в ней только нет, замедление времени от скорости движения и вблизи массивных тел, как следствие парадокс близнецов, эйнштейн утверждал что при приближении к скорости света время замедляется, при достижении ее оно на объекте остановится, а при превышении время потечет назад, разгоняемое тело сплюснется до нуля и исчезнет, И ЭТО ТОЛЬКО ЦВЕТОЧКИ. Все ЭТО построено на одном глюке который у эйнштейна произошел в юности, изучая его биографию я понял откуда ноги растут.
Проанализируйте то что написано ниже в вы поймете в чем проблема и ЭТО все публикуется в официальных учебниках и вдалбливается как истина.
Согласно Ньютону, обогнать свет можно без особого труда — ведь ни сам свет, ни его скорость не представляют собой ничего особенного. Это означало, что если вы будете нестись рядом с лучом света со скоростью, равной его скорости, то луч в вашей системе координат остановится. Но Эйнштейн еще в юности понял, что никто никогда не видел неподвижной световой волны — и вообще непонятно, как ее можно остановить. А значит, решил он, механика Ньютона здесь не работает.
А все зло в эйнштейновских системах отчета.
Согласно представлениям людей едущую машину можно обогнать без труда – ни сама машина ни ее скорость не представляет ничего особенного. Это означало, что если вы будете нестись рядом с машиной со скоростью равной ее скорости, то машина в вашей системе координат остановится. Но Эйнштейн еще в юности понял, что никто никогда не видел неподвижной машины у которой на спидометре 60км.час. А значит, решил он, законы физики здесь не работают. (САРКАЗМ)
Время замедляется, останавливается, идет назад, потому что так написано в формуле, объекты сжимаются и исчезают потому что так написано в формуле, масса растет до бесконечности потому что так написано в формуле, в черных дырах время стоит и тело будет падать на нее вечно потому что так написано в формуле, полностью ПУСТОЕ пространство искривляется, а потом оказывается что оно вообще плоское.
Больше всего доставляет глюки со временем и закон причинности обязательно его прочтите далее



Для начала напомним основные аспекты, относящиеся к проблеме скорости света. Прежде всего: почему нельзя (при обычных условиях) превысить световой предел? Потому, что тогда нарушается фундаментальный закон нашего мира — закон причинности, в соответствии с которым следствие не может опережать причину. Никто никогда не наблюдал, чтобы, например, сначала замертво упал медведь, а потом выстрелил охотник. При скоростях же, превышающих с, последовательность событий становится обратной, лента времени отматывается назад. В этом легко убедиться из следующего простого рассуждения.

Предположим, что мы находимся на неком космическом чудо-корабле, движущемся быстрее света. Тогда мы постепенно догоняли бы свет, испущенный источником во всё более и более ранние моменты времени. Сначала мы догнали бы фотоны, испущенные, скажем, вчера, затем — испущенные позавчера, потом — неделю, месяц, год назад и так далее. Если бы источником света было зеркало, отражающее жизнь, то мы сначала увидели бы события вчерашнего дня, затем позавчерашнего и так далее. Мы могли бы увидеть, скажем, старика, который постепенно превращается в человека средних лет, затем в молодого, в юношу, в ребёнка… То есть время повернуло бы вспять, мы двигались бы из настоящего в прошлое. Причины и следствия при этом поменялись бы местами.

Хотя в этом рассуждении полностью игнорируются технические детали процесса наблюдения за светом, с принципиальной точки зрения оно наглядно демонстрирует, что движение со сверхсветовой скоростью приводит к невозможной в нашем мире ситуации. Однако природа эйнштейн поставила ещё более жёсткие условия: недостижимо движение не только со сверхсветовой скоростью, но и со скоростью, равной скорости света, — к ней можно только приближаться. Из теории относительности следует, что при увеличении скорости движения возникают три обстоятельства: возрастает масса движущегося объекта, уменьшается его размер в направлении движения и замедляется течение времени на этом объекте (с точки зрения внешнего „покоящегося“ наблюдателя). При обычных скоростях эти изменения ничтожно малы, но по мере приближения к скорости света они становятся всё ощутимее, а в пределе — при скорости, равной с, — масса становится бесконечно большой, объект полностью теряет размер в направлении движения и время на нем останавливается. Поэтому никакое материальное тело не может достичь скорости света. Такой скоростью обладает только сам свет! (А также „всепроникающая“ частица — нейтрино, которая, как и фотон, не может двигаться со скоростью, меньшей с.)

Почему это так? Потому, что препятствием для передачи любого сигнала со скоростью больше с служит всё тот же закон причинности. Представим себе такую ситуацию. В некоторой точке А световая вспышка (событие 1) включает устройство, посылающее некий радиосигнал, а в удалённой точке В под действием этого радиосигнала происходит взрыв (событие 2). Понятно, что событие 1 (вспышка) — причина, а событие 2 (взрыв) — следствие, наступающее позже причины. Но если бы радиосигнал распространялся со сверхсветовой скоростью, наблюдатель вблизи точки В увидел бы сначала взрыв, а уже потом — дошедшую до него со скоростью с световую вспышку, причину взрыва. Другими словами, для этого наблюдателя событие 2 совершилось бы раньше, чем событие 1, то есть следствие опередило бы причину.


. В частной относительности получается, что если частица движется быстрее света в одной системе отсчета, то в другой она может двигаться назад во времени. Иными словами, СС движение или перенос информации автоматически означают путешествие назад во времени или отправку сообщений в прошлое. Если такое возможно, то можно было бы отправиться назад во времени и изменить ход истории, например, убив собственного дедушку. Это и вправду весьма серьезный аргумент, но он оставляет нам возможность соврешать ограниченные СС путешествия при условии, что у нас нет возможности вернуться. Это не очень хорошо. Из относительности следует, что все, что можно сделать в одной системе отсчета, должно быть можно сделать и в другой. Либо может быть так, что путешествия во времнеи возможны и что причинность нарушается определенным образом при СС путешествии.

Моя интерпретация этого всего – Предположим, что мы находимся на неком космическом чудо-корабле, движущемся быстрее света. Предположим что мы догнали радиопередачу радиоволны которой были испущены 10 лет назад, мы услышали прошлое НО ЭТО ЖЕ НЕ ЗНАЧИТ ЧТО МЫ ПЕРЕНЕСЛИСЬ В ПРШЛОЕ.
Я не буду перечислять здесь все то что я наопровергал за месяц, но самое главное на чем построена вся теория относительности я расскажу.

Материал википедии-

Гравитационное красное смещение, или замедление времени в гравитационном поле
Ослабление энергии света, излучаемого звёздами с сильной гравитацией, было предсказано Джоном Митчеллом ещё в 1783 году, на основе корпускулярного представления о свете, которого придерживался Исаак Ньютон. Влияние гравитации на свет исследовали в своё время Пьер-Симон Лаплас и Иоганн Георг ван Зольднер (1801) задолго до того, как Альберт Эйнштейн в статье 1911 года о свете и гравитации вывел свой вариант формулы для этого эффекта.
Филипп Ленард обвинил Эйнштейна в плагиате за то, что он не процитировал более раннюю работу Зольднера — однако, принимая во внимание, насколько эта тема была забыта и заброшена до того момента, как Эйнштейн вернул её к жизни, практически не подлежит сомнению, что Эйнштейн не был знаком с предыдущими работами. В любом случае, Эйнштейн пошёл намного дальше своих предшественников и показал, что ключевым следствием из гравитационного красного смещения является гравитационное замедление времени. Это была очень оригинальная и революционная идея. Эйнштейн впервые предположил, что потерю энергии фотоном при переходе в область с более высоким гравитационным потенциалом можно объяснить через разность хода времени в точках приёма и передачи сигнала. Энергия кванта электромагнитного излучения пропорциональна его частоте согласно формуле где — постоянная Планка. Таким образом, если время для приёмника и передатчика течёт с разной скоростью, наблюдаемая частота излучения, а вместе с ней и энергия отдельных квантов, тоже будет различной для приёмника и передатчика.

Свет идущий от массивных объектов испытывает красное смещение, но почему. На следующем рисунке изображена земля ее гравитационное поле
http://s004.radikal.ru/i208/1006/fc/979b8e2307fc.jpg (http://www.radikal.ru)

Теория относительности же утверждает что длинна волны увеличивается потому время для приемника №2 и источника света (земля) время течет с разной скоростью, ЧТО МЫ ВИДИМ ЗАМЕДЛЕННЫЙ СВЕТ №4, а эйнштейн не подумал что мы будучи в точке №2 видим свет не тот который сейчас замедленный, а тот который до нас дошел №5 – НЕЗАМЕДЛЕННЫЙ. По сути никакого красного смещения быть не должно, А ОНО ЕСТЬ, и замедление времени КОТОРОГО НЕТ не причем.
Под цифрой 3 показано что реально происходит со светом длинна его волны увеличивается – свет теряет энергию.
И это утверждение на котором построена вся современная физика, как по мне физика в МЕ с ее межзвездными полетами и масс полями намного реальней нашей, с возвратом и остановкой времени и кривым плоским пространством.

А математически можно описать и плоскую землю стоящую на троих слонах, , при этом понадобится ввести 126 измерений и появятся сингулярности в области плавания черепахи – что мы сейчас и наблюдаем в теории струн.
Один психолог сказал, что если бы у людей не било религии то их молитвой было бы ЕЕЕЕЕЕЕЕ РААААВВВНО ЭЭЭЭЭЭММ ЦЕЕЕЕ КВАДРАААТ, ВОИСТИНУ КВАДРААТ.

Сейчас в науке настал период подобный средневековью, где все, что противоречит взглядам релятивистов сжигается и никогда не публикуется, тормозя развитие науки.
На просторах интернета уже есть несколько человек, чьи выводы совпадают с моими. В конце своих изучений, я нашел книгу автор, которой уже ничего и не опровергает, а уже более 10 лет создает свою единую теорию. И я думаю за этой теорией будущее. Я может еще и в космос слетаю...
http://s46.radikal.ru/i114/1006/57/4584133001aa.jpg (http://www.radikal.ru)

Вот очень интересная инфа по поводу масс технологий
http://blog.kp.ru/users/neferit/post119936076/
http://anastgal.diary.ru/p75328629.htm

Нафлудил я тут прилично, что ж поделаешь - люблю писать статьи.:sml:

У радио нет будущего. Летательные аппараты тяжелее воздуха невозможны. Скоро выяснится, что рентгеновские лучи - мистификация.
Лорд Кельвин, физик, 1899 г.

А прошло всего лишь 100 лет.

Dragon27
21.06.2010, 20:25
Почему у нас тут нету раздела специально для таких вот альтернативщиков? Обучать их уже бесполезно, вот пускай и варятся в собственном соку.

luden
22.06.2010, 11:08
Dragon27, ну, вот что мы будем без них делать? Обложимся учебниками и будем дорабатывать квантовую физику?
iLLi-DAN, Как известно из теории относительности тело не может достичь скорости света потому что его масса становится бесконечно большой при приближении к С
Нет. При превышении скорости света ты вылетаешь в комплексные плоскости. Соответственно, дальнейшее рассуждение бессмыслено.

Zosia65
22.06.2010, 16:02
А вот пускай это и будет таким разделом. Давайте посмотрим на графическое изображение ЭМ кванта. Всплеск электрического поля, где же магнитное? Далее всплеск магнитного, а где же электрическое? Между ними вообще ничего... Взаимопорождающие поля, существующие одновременно лишь по отдельности, с сингулярностью между собой. Очевиден процесс более наших привычных трех измерений. При за световых скоростях решения в комплексных плоскостях? А их нельзя применить на до световых? Потом просто свести, так сказать к частному случаю, Ньютону и Евклиду.
iLLi-DAN, Исчезновение массы в одном месте и появление в другом нарушит законы причинности. Масса должна быть перемещена, что бы не нарушить связи со ВСЕМИ гравитирующими объектами. Любая масса гравитирует со ВСЕМИ объектами. Все объекты(!) гравитируют с ЛЮБОЙ массой. Впрочем решение возможно. Квантовый переход массы на энергетически заданное расстояние... Она не исчезнет на время перехода, и все тип топ. Только это не мгновенно.

В ОТО есть непоняточка, стремление массы к бесконечности при световых скоростях. Значит и энергия стремится к бесконечности. Тогда каков смысл эм це квадрат? Еще и время тормозит. Имхо, когда тело проходит энергию Мдвиж(V)^2=Мпокоя(С)^2, оно проходит момент сингулярности. Далее оно находится за горизонтом событий, и свет от него нас никогда не достигнет.
И еще. ЧД не теряет массу. А значит гравитация распространяется со скоростью, как минимум выше скорости света. Отсюда,- скорость света не предел. Иначе: Наступило мгновение наступления горизонта событий, или просто ЧД. Свет уже не может вырваться из этих объятий. И... Стала разваливаться система в которой эта масса была ядром. Ведь и распространение гравитации ограничено скоростью света.:)

luden
22.06.2010, 16:25
А их нельзя применить на до световых?
А зачем? Мнимые числа не зря назвали мнимыми. Если мы говорим о принципиальной возможности движения выше скорости света, то тут нужно дорабатывать/опровергать СТО.
В ОТО есть непоняточка, стремление массы к бесконечности при световых скоростях.
Вот я формулы это подтверждающей найти не могу. Чего-то выглядит это заявление слишком парадоксально даже для ОТО.
Наступило мгновение наступления горизонта событий, или просто ЧД. Свет уже не может вырваться из этих объятий. И... Стала разваливаться система в которой эта масса была ядром. Ведь и распространение гравитации ограничено скоростью света.
Не очень понял. Мы имеем некий начальный диаметр гравиполя, который начинает расширятся. Чем это вредит самой ЧД находящейся в центре этого радиуса. В чём развал системы? Фотоны всё так же не могут выбраться и тд.

<KANE>
22.06.2010, 16:50
с этим эффектом связана ошибка в работе оружия). Допустим человечество все таки смогло создать корабль который бы разгонялся до сверхсветовой КАК ТОРМОЗИТЬ? В МЕ эта проблема отсутствует. Насчет оружия, в МЕ обычные гауски снабжаются маленьким масс генератором который на участке ствола создает поле с пониженной массой что позволяет разогнать снаряд до бешенных скоростей но со снарядом вылетевшим из масс поля произойдет то же самое что и с нормандией при отключении масс поля он резко затормозится, такая вот фишка – недоглядели.
Ну хорошо, а что делать с тем фактом что масса снаря выпускаемого из винтовки крайне мала? В книгах эти снаряды описывается так что их размер не больше бусинки. Следовательно и масса этого снаряда очень мала. Она же не имеет вес сравнимый с весом космического корабля. Да и в открытом космосе вес не имеет особого значения если только предмет не приближается к планете, а если это случилось то значит этот предмет начинает набирать вес за счёт действия гравитационного поля планеты.

Dragon27
22.06.2010, 17:18
В ОТО есть непоняточка, стремление массы к бесконечности при световых скоростях.
В СТО, а не ОТО. И не массы, а
энергия стремится к бесконечности
То есть для того, чтобы разогнать тело до состояния с бесконечной кинетической энергией, нужно передать телу бесконечную энергию. И всё. Разогнать до скорости света нельзя, потому что мы не можем передать тело бесконечное количество энергии. Просто и понятно.

Тогда каков смысл эм це квадрат?
Энергия покоя тела равна инвариантной массе. С точностью до множителя це-квадрат.

Имхо, когда тело проходит энергию Мдвиж(V)^2=Мпокоя(С)^2, оно проходит момент сингулярности.
Что вы так прицепились к этой сингулярности, что она у вас изо всех щелей лезет?

ЧД не теряет массу.
Теряет :)

А значит гравитация распространяется со скоростью, как минимум выше скорости света.
Откуда вывод?

Отсюда,- скорость света не предел.
Выше я давал 2 ссылки, в которым чёрным по белому объясняется, что, конечно же, не предел. В ОТО.

Добавлено через 1 минуту
Dragon27, ну, вот что мы будем без них делать? Обложимся учебниками и будем дорабатывать квантовую физику?
Одно дело - не знающие люди. С ними ещё можно общаться, объяснять.
Другое дело - воинственные неучи.

Zosia65
23.06.2010, 11:07
Dragon27, я говорю о моменте появления ЧД. Наступление горизонта событий, ни на йоту не снижает гравитацию ЧД. А вот свет из своих недр обрезает.
А может растет масса системы ЧД и падающей в нее звезды? Вещество звезды при падении значительно ускоряется, растет его масса... Бах, после горизонта событий осталась масса одной ЧД. Тупо? Да, если не принять скорость гравитационных взаимодействий, как минимум выше скорости света.
m=m0/sqrt(1-(v^2/c^2)),
где sqrt - квадратный корень,
m0 - масса покоя,
v^2 - квадрат скорости движения,
c^2 - скорость света в квадрате.
При v равной c имеем бесконечность. Вот к этой гадости я и цепляюсь, обзываю сингулярностью, предполагаю скачок - за V=С минус дельта, скачок до V=С плюс дельта. Пускай Вас не пугает мнимая масса, масса та же, появляется пространство с мнимой кривизной. Такая математика прекрасно работает: например, индуктивное или емкостное сопротивление в электротехнике полагается величиной чисто мнимой. И ничего - все цепи прекрасно рассчитываются и работают в соответствии с этими самыми расчетами в комплексных числах. Просто в натуре сопротивлению sqr(-1) соответствует фазовый сдвиг между током и напряжением в 90 градусов. А в расчетах оказывается удобнее использовать операторный метод, а не тригонометрию...

luden
23.06.2010, 11:33
Наступление горизонта событий, ни на йоту не снижает гравитацию ЧД.
Стоп. То есть ты считаешь, что в случае распространения гравитации со скоростью ниже световой, её замкнёт на себя в ЧД и она не сможет выйти за горизонт событий?
Такая математика прекрасно работает: например, индуктивное или емкостное сопротивление в электротехнике полагается величиной чисто мнимой.
Да, вот только есть разница между графическим решением, в котором мнимые числа всего лишь смена оси, и физическим, в котором мнимые числа это перпендикулярная вселенная.

Dragon27
23.06.2010, 17:00
Dragon27, я говорю о моменте появления ЧД.
С точки зрения внешнего наблюдателя этот момент никогда не наступает. Но какая разница, тяготение почти-почти такое же, как в ЧД (и приближается к нему), и объект всё равно остаётся чёрным, а свет из него так и не выбирается. С точки зрения внешнего наблюдателя.
И при чём тут скорость гравитации вообще?

m=m0/sqrt(1-(v^2/c^2))
Забудь ты про эту формулу, релятивистской массы уже давно нету в современной физике.

При v равной c имеем бесконечность.
И ни одно тело не может добраться до v равному c. В чём проблема? Никакой сингулярности никто не достигает.

Zosia65
24.06.2010, 11:43
ЭМ излучение, это колебания распространяющиеся в пространстве,- колебания ПРОСТРАНСТВА электрическими=магнитными полями. Гравитация,- искривление ПРОСТРАНСТВА (напрашивается не полная аналогия, с помещающимся в жидкости телом). И еще одно измерение для этого искривления уже имеется,- ВРЕМЯ. Искривление пространства, НЕ МОЖЕТ привести к каким то дополнительным искривлениям для воздействия на самое себя. Т.е., гравитационные взаимодействия, не подвластны создаваемым ими релятивистским эффектам.


Dragon27, тело не достигнет, кто спорит. Только какая бесконечность? Горизонт событий, это момент искривления пространства, с эффектом эквивалентным скорости света. Дальше идет ПРЕВЫШЕНИЕ. Сингулярность, это не весь объем ЧД. Это сфера вокруг нее, с нулевой толщиной (дайте ей хоть какую то толщину, и сможете наблюдать стоячую ЭМ волну :), посчитайте ее точкой, получите нулевой объем ЧД, исключите ее, и забудьте о самой ЧД).
До нее у тела скорость меньше световой, после - больше. Момент перехода, это Е=Мотн.V^2=Мпок.С^2. Верните релятивистскую массу, это относительно наблюдателя! Для него, внутренности ЧД, это мнимое пространство мнимых объектов. Хотя и он сам для них мнимый, со всем своим пространством.

luden
24.06.2010, 12:49
Искривление пространства, НЕ МОЖЕТ привести к каким то дополнительным искривлениям для воздействия на самое себя.
Угу, тогда я не понимаю, что ты описываешь.

Dragon27
24.06.2010, 22:17
ЭМ излучение, это колебания распространяющиеся в пространстве,- колебания ПРОСТРАНСТВА электрическими=магнитными полями.
Что-то ты не то сказанул.

Горизонт событий, это момент искривления пространства, с эффектом эквивалентным скорости света. Дальше идет ПРЕВЫШЕНИЕ.
Вот только СТО к этому уже не имеет никакого отношения.

Сингулярность, это не весь объем ЧД. Это сфера вокруг нее, с нулевой толщиной
Нет, это гипотетическая точка в центре ЧД.

и сможете наблюдать стоячую ЭМ волну
Стоячую ЭМ волну и без ЧД можно спокойно наблюдать при отржениях.

До нее у тела скорость меньше световой, после - больше. Момент перехода, это Е=Мотн.V^2=Мпок.С^2. Верните релятивистскую массу, это относительно наблюдателя! Для него, внутренности ЧД, это мнимое пространство мнимых объектов. Хотя и он сам для них мнимый, со всем своим пространством.
Ещё раз тебе говорю, запомни это хорошенько - СТО здесь совершенно не применима, никаких мнимых поверхностей, сингулярностей и прочего на ГС нету. ГС - условная граница невозвратимости. Никакими спецэффектами в пространстве он не выделен.

Zosia65
25.06.2010, 10:55
Dragon27, ЭМ волна, это колебания чего? В чем? Какой умник назвал точку горизонтом, условной границей (С ЧЕМ?)? Скорость света постоянна и не зависит от скорости источника-приемника, какая стоячая ЭМ волна? А Вы сами понимаете вопрос? Вы уверены, что оспариваете суть моего предположения? СТОшка предсказывает наличие ЧД, однако не применима для их рассмотрения... Теория струн, вложенность параллельных миров, белые дыры... Да, сией ереси нет ни в СТО, ни в ОТО. Это истина в последней инстанции? Аминь.

Dragon27
25.06.2010, 14:32
ЭМ волна, это колебания чего?
ЭМ поля. И не совсем колебания, под колебаниями обычно подразумевают не распространяющееся в пространстве изменение величины.

В чем?
Любое поле заполняет всё пространство.

Какой умник назвал точку горизонтом, условной границей (С ЧЕМ?)?
Не точку, а сферу. А в 4D другую фигуру.

Скорость света постоянна и не зависит от скорости источника-приемника, какая стоячая ЭМ волна?
Какая стоячая ЭМ волна? Волны Шумана, например.

Вы уверены, что оспариваете суть моего предположения?
А в чём суть вашего предположения?

СТОшка предсказывает наличие ЧД, однако не применима для их рассмотрения...
СТОшка тут вообще не при чём. ЧД - порождение и рассмотрение ОТО.

Теория струн, вложенность параллельных миров, белые дыры... Да, сией ереси нет ни в СТО, ни в ОТО. Это истина в последней инстанции?
Теория струн, параллельные миры и белые дыры вы, конечно, можете обсуждать (только это гораздо сложнее, чем кажется). Но при чём тут они? Мы пока про белые дыры ни слова ни сказали, вы пока про СТО и горизонты событий что-то пытались высказать.
И, да, СТО и ОТО в отличии от теории струн и прочего, экспериментально подтверждены.

Аминь.
Обычно ставится после фраз или утверждений, а не вопросов.

iLLi-DAN
26.06.2010, 22:12
(Пополнил счет на интернет и вернулся)
Вот так всегда, посредством форума невозможно ничего нормально объяснить, когда общаешься с людьми вживую тебя понимают слету, а тут такое дело. Я уже не знаю куда проще объяснять и что бы выложить здесь все что я наопровергал мне потребуется как минимум день работы (на первый пост я итак убил 2 часа).

Zosia65
iLLi-DAN, Исчезновение массы в одном месте и появление в другом нарушит законы причинности. Масса должна быть перемещена, что бы не нарушить связи со ВСЕМИ гравитирующими объектами Квантовый переход массы на энергетически заданное расстояние... Она не исчезнет на время перехода, и все тип топ. Только это не мгновенно.

Я говорю не про Квантовый переход массы, а совершенно про другое
А кто сказал что масса исчезает –
Давайте уясним что наделяет обекты массой – массы нет, есть сопротивление вещества всемирному полю,(подобно шарику опущенному в воду) работа ретранслятора сродни установке в воде пластиковой трубы в которую вода не просачивается и ТОМУ шарику, что перемещается в такой трубе (передавать ему ускорение становится намного легче) ретрансляторы работают так же.

Я не понимаю неужели вы ЭТО считаете ЗАКОНОМ причинности ну как ЭТО ВООБЩЕ можно считать законом.перечитайте НУ ВДУМАЙТЕСЬ!!!

Для начала напомним основные аспекты, относящиеся к проблеме скорости света. Прежде всего: почему нельзя (при обычных условиях) превысить световой предел? Потому, что тогда нарушается фундаментальный закон нашего мира — закон причинности, в соответствии с которым следствие не может опережать причину. Никто никогда не наблюдал, чтобы, например, сначала замертво упал медведь, а потом выстрелил охотник. При скоростях же, превышающих с, последовательность событий становится обратной, лента времени отматывается назад. В этом легко убедиться из следующего простого рассуждения.
Предположим, что мы находимся на неком космическом чудо-корабле, движущемся быстрее света. Тогда мы постепенно догоняли бы свет, испущенный источником во всё более и более ранние моменты времени. Сначала мы догнали бы фотоны, испущенные, скажем, вчера, затем — испущенные позавчера, потом — неделю, месяц, год назад и так далее. Если бы источником света было зеркало, отражающее жизнь, то мы сначала увидели бы события вчерашнего дня, затем позавчерашнего и так далее. Мы могли бы увидеть, скажем, старика, который постепенно превращается в человека средних лет, затем в молодого, в юношу, в ребёнка… То есть время повернуло бы вспять, мы двигались бы из настоящего в прошлое. Причины и следствия при этом поменялись бы местами.
От того что они догнали бы волны старой радиопередачи ВРЕМЯ БЫ ПОШЛО НАЗАД как можно верить в это, именно верить понимают не очень многие.

<KANE>
Ну хорошо, а что делать с тем фактом что масса снаря выпускаемого из винтовки крайне мала? В книгах эти снаряды описывается так что их размер не больше бусинки. Следовательно и масса этого снаряда очень мала. Она же не имеет вес сравнимый с весом космического корабля. Да и в открытом космосе вес не имеет особого значения если только предмет не приближается к планете, а если это случилось то значит этот предмет начинает набирать вес за счёт действия гравитационного поля планеты.

Ну я не совсем понял твой вопрос? говори конкретно, но насколько я понял

Ты путаешь массу с весом. Давай порассуждаем – в открытом космосе веса вообще нет, вес создает гравитационное поле планет и звезд, Она же не имеет вес сравнимый с весом космического корабля , я уже писал что если снаряд маленький (бусинка) то никакой массгенератор в гауске не понадобится, но зачем они его туда ставят. Вот как действует масс генератор, он снижает массу, НЕ ВЕС, веса в открытом космосе нет, а если бы снижал вес, то это ничего не дало бы ( ведь в невесомости пуля выпущенная из пистолета не летит быстрее). Снижается масса, тоесть корабль становится не 50 000 тонн, а 5 грамм, и что бы его разогнать допустим до 50 метров в секунду хватит маленькой петарды, а уж двигателя корабля хватит на огого.

в МЕ обычные гауски снабжаются маленьким масс генератором который на участке ствола создает поле с пониженной массой что позволяет разогнать снаряд до бешенных скоростей ( и на разгонку этой бусинки «10 грамм» уйдет столько энергии, столько ушло бы на разгонку скажем протона так как масса бусинки уже не 10 г в этом поле а 0,0000000000001 ) но со снарядом вылетевшим из масс поля произойдет то же самое что и с нормандией при отключении масс поля он резко затормозится, подумай снаряд разгоняется в разных средах, это все равно что сказать что бросив камень рукой, предав ему скорость допустим 10 м.сек (разгонялся камень в среде воздуха) что если такой камень попадет в воду( выйдет из масс поля) то скорость его движения в воде будет такой же как в воздухе.
Я уже так устал спорить с релятивистами, посредством форума сложно что то доказать, я уже наспорился на форумах по физике, живой собеседник же сразу все понимает.

Luden
Нет. При превышении скорости света ты вылетаешь в комплексные плоскости. Соответственно, дальнейшее рассуждение бессмыслено.

Это как , можно поподробнее?

Вот как мне сначала все объясняли -
Итак, тело нельзя разогнать до скорости света потому что: при приближении тела к скорости света ускорение тела уменьшается до нуля какая ни была бы ускоряющая сила
- почему?
потому что: при приближении тела к скорости света его масса возрастает до бесконечности (энергия прямопропорциональна массе), а чтобы разогнать тело с бесконечной массой нужна бесконечная энергия.
- а почему масса возрастает до бесконечности?
Ну возрастает не масса как такова, а инерционная масса то бишь инерция
- так почему же возрастает до бесконечности инерция?
Ну увеличение инертности при больших скоростях мы объяснили уменьшением ускорения при большой скорости
- ТАК ПОЧЕМУ ЖЕ уменьшается ускорение?
Потому что увеличивается инерция.
И так объясняется везде, у попа была собака...
Вот вам картинка-
http://s49.radikal.ru/i126/1006/a9/699cdef35433.jpg (http://www.radikal.ru)

Как я уже сказал сюда я пришел не копья ломать, а помечтать и порассуждать и найти единомышленников. Кому действительно интересно почему время не замедляется и не ускоряется и не может, останавливается (время это единица измерения действия, как килограмм), что является истинной причиной грави красного смещения, почему нет сингулярностей и прочих пошлостей,горизонта событий несушествует и т.д. Рекомендую прочесть вышеуказанную книгу(плевая физика) это вторая действительно стоящая вещь вместе с книгой Хокинга «краткая история времени» ( именно она и масс натолкнули меня на все опровержения) в интернете много хлама но эти действительно классные. У меня ушло на полевую 2 дня, не поленитесь и без предвзятостей возьмите и прочитайте от начала до конца, много чего поймете.:sml:

Dragon27
27.06.2010, 00:35
Я уже так устал спорить с релятивистами, посредством форума сложно что то доказать, я уже наспорился на форумах по физике, живой собеседник же сразу все понимает.
Как правило, живой собеседник (в отличие от посетителя физического форума) имеет гораздо меньшую вероятность разбираться в теории относительности. Именно поэтому не разбирающийся человек быстрее поддаётся заблуждению.

Итак, тело нельзя разогнать до скорости света потому что: при приближении тела к скорости света ускорение тела уменьшается до нуля какая ни была бы ускоряющая сила
Что-то вроде того.

- почему?
потому что: при приближении тела к скорости света его масса возрастает до бесконечности (энергия прямопропорциональна массе)
Неверно. Ни первая часть предложения, ни вторая. Импульс (и кинетическая энергия) возрастают до бесконечности. Ну и инертность, да.

Ну возрастает не масса как такова, а инерционная масса то бишь инерция
- так почему же возрастает до бесконечности инерция?
Ну увеличение инертности при больших скоростях мы объяснили уменьшением ускорения при большой скорости
- ТАК ПОЧЕМУ ЖЕ уменьшается ускорение?
Потому что это (релятивистская динамика) простое следствие преобразований Лоренца. Не создавайте сами себе замкнутых кругов.

Можете самостоятельно использовать, например, формулу изменения скоростей при смене систем отсчёта, для просчёта скорости относительно лабораторной системы отсчёта при постоянном ускорении относительно СО ускоряемого тела.
И убедиться.

Ваша проблема в том, что ваши источники ограничиваются википедией и второсортной литературкой (например, научно-популярной).

Нужные доказательства по данному вопросу вы можете почерпнуть, например, в Тейлор, Уилер "Физика пространства-времени". Или Мизнер, Торн, Уилер "Гравитация", глава 6-ая "Ускоренные наблюдатели", $6.2 "Гиперболическое движение."

Как я уже сказал сюда я пришел не копья ломать, а помечтать и порассуждать и найти единомышленников.
Увы, одних мечтаний для того, чтобы досконально разобраться хотя бы в СТО недостаточно.

У меня ушло на полевую 2 дня, не поленитесь и без предвзятостей возьмите и прочитайте от начала до конца, много чего поймете.
И как такое можно воспринимать серьёзно? Каких-то жалких 2 дня.

luden
27.06.2010, 10:35
Это как , можно поподробнее?
Обрати внимание на формулы. При превышении скорости света там появляется корень отрицательных значений. Это вынуждает нас пользоваться мнимыми числами для дальнейших расчётов. Если очень упрощённо, то геометрически это смена оси координат в двухмерном пространстве, корень из минус единицы по оси Х даст единицу по оси Y. Так вот если мнимые координаты это ещё куда ни шло, то мнимая масса перпендикулярная обычной вещь выходящая за пределы допуска. Таким образом, при теоретическом превышении скорости света мы получаем противоречивую сущность, что ставит нас в ситуацию, что на принятые на сегодняшний день теории выход за скорость света - невозможен.
Рекомендую прочесть вышеуказанную книгу(плевая физика)
Цитирую автора
Полевая физика – это новый подход к пониманию природы вещей и устройства Мироздания. По сути, это самостоятельная ветвь физической науки, свои философия и методология, механика и математика. В отличие от традиционной науки, в Полевой физике основное внимание уделяется не описательному подходу, отвечающему только на вопросы "Как?", а поиску внутренних механизмов физических явлений и объяснению их природы, то есть ответам на вопросы "Почему?". В Полевой физике пробретают новый смысл такие важные понятия, как масса и энергия, пространство время и движение, сила и скорость, а также взаимодействия и физические поля.
Позвольте я восприму это как эпиграф и не буду читать второсортную псевдонаучную мукулатуру.
вместе с книгой Хокинга «краткая история времени»
Хокинг грамотный, но пишет не очень правильно, я считаю. После его прочтения неизбежно возникает иллюзия того, что ты всё понял, хотя это и не так.

Dragon27
27.06.2010, 12:38
Р. Пенроуз "Путь к реальности". Эта книга со стороны кажется тем же научпопом, но на самом деле это очень серьёзная книга (практически серьёзный учебник). Советую изучить.

Zosia65
27.06.2010, 13:17
Dragon27, почему сингулярность, это точка где то внутри? Обычная и сверхмассивная ЧД имеют идентичные точки? Давайте рассмотрим горизонт событий. Это место, в котором искривление пространства достигает такой величины, что тело помещенное в него, УЖЕ ИМЕЕТ СКОРОСТЬ СВЕТА, относительно внешнего наблюдателя. И тело то не причем, это уже пространство ТАКОЕ. Это как бы невозможно, потому и названо сингулярностью. И дело не конечности скорости света, главное невозможность времени равного нулю! А далее тело испытывает еще большие гравитационные воздействия. Ничего не запрещает ему иметь большую скорость. Математика продолжает работать. Повторюсь. ЧД, это не просто сила, стремящаяся разогнать тело до скорости света, и вечное падение к центру (время стремится остановиться). ТЕЛО УЖЕ ОКАЗЫВАЕТСЯ В СОСТОЯНИИ ИДЕНТИЧНОМ СКОРОСТИ СВЕТА, ПРИ ПЕРЕСЕЧЕНИИ ГОРИЗОНТА СОБЫТИЙ. Там такие свойства пространства! Впрочем, я готов подождать, пока Вы это вычитаете в источниках пользующихся у Вас доверием.

iLLi-DAN
27.06.2010, 13:27
И как такое можно воспринимать серьёзно? Каких-то жалких 2 дня.

Я так понял вам жалко Каких-то жалких 2 дня? ну тогда уделите 10 минут этому http://n-t.ru/tp/ns/ol.htm и скажите свое мнение.


Спасибо за рекомендации книг (трудно найти что-то хорошее) буду читать.

Но если вы так прекрасно во всем разбираетесь, объясните грави-красное смещение, и серьезно вы действительно считаете что в ЧД время стоит на месте и они обладают бесконечной массой и плотностью.

luden
Позвольте я восприму это как эпиграф и не буду читать второсортную псевдонаучную мукулатуру.

И почему люди судят о книге по первому абзацу, не утруждая себя почитать хоть пол дня, и называют мукулатурой потому что автор назвал ее самостоятельной ветвью физики.:frown:

По крайней мере он пытается дать конструктивные ответы на вопросы, на которые современная физика отвечает просто НЕ ЗНАЮ.

Zosia65
27.06.2010, 14:07
iLLi-DAN, экранируя тело от гравитационных воздействий, Вы исключаете его из Вселенной. Даже ЧД, продолжают иметь массу. Потом это тело опять вводите в систему... Давайте уж начнем с создания перпетуум мобиле. Перпетуум пожиратель по понятным причинам делать не будем.

luden, после горизонта событий в ЧД, начинается пространство с мнимой для нас кривизной. Наша Вселенная, это ЧД в сверх Вселенной. Все что падает за нами, с ускорением продолжает отставать, испытывая воздействие позже. Мы воспринимаем сие собственным все ускоряющимся расширением (черная масса и черная энергия, факт из нашей родительской Вселенной). Имхо.

Добавлено через 17 минут
Dragon27, и еще о сингулярности. Следствие гравитации? Любое тело в центре масс любого количества, находится в невесомости, так как гравитационные воздействия взаимокомпенсируются в любую сторону! На тело действуют только силы молекулярного (атомного, ядерного...) давления. Отчего сингулярность в центре?

Dragon27
27.06.2010, 14:44
Dragon27, почему сингулярность, это точка где то внутри?
По ОТО в центре ЧД материя достигает бесконечной плотности (конечно это не так, ведь там в действие вступают другие эффекты ;) ) и нигде более.
Но вы правы, у некоторых ЧД (вращающиеся, прочие) сингулярность может быть сосредоточена не только в точке, но и, например, в линии. Но всё же внутри, в центре.

Давайте рассмотрим горизонт событий. Это место, в котором искривление пространства достигает такой величины, что тело помещенное в него, УЖЕ ИМЕЕТ СКОРОСТЬ СВЕТА, относительно внешнего наблюдателя. И тело то не причем, это уже пространство ТАКОЕ. Это как бы невозможно, потому и названо сингулярностью
Нет, не верно. Возможно. Этим ОТО и отличается от СТО. И сингулярностью называть это терминологически неверно. И здесь не применимы преобразования Лоренца. Это не пекулярная скорость, это скорость "движения" пространства.
Поймите - это не СТО, это ОТО. В СТО плоское пространство-время. В ОТО оно имеет псевдориманову метрику.

Я так понял вам жалко Каких-то жалких 2 дня?
На прочтение бреда - да. Я лучше потрачу пару лет на штудирование лекций Фейнмана с задачником и изучение университетских общих курсов по физике.

объясните грави-красное смещение
Космологическое красное смещение?

вы действительно считаете что в ЧД время стоит на месте и они обладают бесконечной массой и плотностью.
Нет.

Следствие гравитации? Любое тело в центре масс любого количества, находится в невесомости, так как гравитационные воздействия взаимокомпенсируются в любую сторону! На тело действуют только силы молекулярного (атомного, ядерного...) давления. Отчего сингулярность в центре?
Гравитация в ОТО отличается от Ньютоновской. Это надо изучать и очень долго.
ЧД сама себя стягивает с огромной скоростью, как и любое тело. Только обычное тело (звезда) умеет расталкивать себя внутренним давлением и прочим. ЧД так не умеет, гравитация слишком сильна. Все внутренности ЧД на огромной скорости влетают в центр тяготения и образуют там огромную плотность. И само гравитационное поле сильным самодействием (это уже КТП) действует на само себя. А там уже квантовые эффекты бла-бла-бла, это ещё не разработано.
К тому же можно математически расписать метрику ЧД и вычислить кривизну в её центре.

ну тогда уделите 10 минут этому http://n-t.ru/tp/ns/ol.htm и скажите свое мнение.
Ладно, после обеда посмотрю.

iLLi-DAN
27.06.2010, 15:22
Космологическое красное смещение?


Нет.

Гравитационное красное смещение
Ослабление энергии света, излучаемого звёздами с сильной гравитацией.

Dragon27
27.06.2010, 15:26
Посмотрел статейку.

Эйнштейн в начале своего вывода преобразования Лоренца повторяет допущение: «пусть x' = x – vt»
В "К электродинамике движущихся тел" я такого не нашёл. Уж извините.
Здесь (http://ru.wikipedia.org/wiki/%D0%92%D1%8B%D0%B2%D0%BE%D0%B4_%D0%BF%D1%80%D0%B5% D0%BE%D0%B1%D1%80%D0%B0%D0%B7%D0%BE%D0%B2%D0%B0%D0 %BD%D0%B8%D0%B9_%D0%9B%D0%BE%D1%80%D0%B5%D0%BD%D1% 86%D0%B0) вы можете просмотреть вывод преобразований Лоренца и попробовать найти ошибку. Статью Эйнштейна вы можете сами найти и попробовать найти ошибку там.

Так что
Исходная посылка и конечный результат в выводе преобразования Лоренца противоречат друг другу.
В выводе преобразований Лоренца замечено не было.

Лоренц и его последователи не исследовали (не искали) класс возможных преобразований, сохраняющих уравнения Максвелла неизменными.
Не имеет значения, так как преобразования Лоренца удовлетворяют и опыту и теории.

Скорость, входящая в преобразование Лоренца, не соответствует действительной (галилеевой) скорости относительного движения инерциальных систем отсчета.
Автор измеряет неверно. У него скорость движения даже зависит от угла зрения, что не придаёт дополнительной ценности его способам измерения.
Простейший метод - пройденное в одной системе отсчёта расстояние поделить на отсчитанное в этой же системе время - показывает совершенно верный результат.

Преобразование Лоренца использовалось для вращательного движения без физического обоснования.
Автор посчитал, что вращающееся тело должно разрушиться. И вывод Эйнштейна об отсутствии абсолютно жёстких тел он почему-то не принял. Вращающееся тело не разрушится, релятивистские явления лишь увеличат продольное растяжение тела. Прочитайте тут (http://ru.wikipedia.org/wiki/%D0%9F%D0%B0%D1%80%D0%B0%D0%B4%D0%BE%D0%BA%D1%81_% D0%AD%D1%80%D0%B5%D0%BD%D1%84%D0%B5%D1%81%D1%82%D0 %B0).

Неверное изложение пространственно-временных отношений, обусловленное гносеологической ошибкой: интерпретацией явлений как сущностей.
Стандартная отговорка, которая подразумевает неизвестно что и давно уже опровергнута опытом.

Абсолютизация преобразования Лоренца, т.е. неправомерное распространение этого преобразования, которое справедливо только для электромагнитных волн, для всех без исключения физических явлений материального мира.
А попросту бы противоречило принципу относительности, так как можно было бы установить абсолютную систему отсчёта с помощью несветовых явлений.

Добавлено через 1 минуту
Гравитационное красное смещение
Ослабление энергии света, излучаемого звёздами с сильной гравитацией.
А что его объяснять, коли оно и так уже объяснено (http://ru.wikipedia.org/wiki/%D0%93%D1%80%D0%B0%D0%B2%D0%B8%D1%82%D0%B0%D1%86%D 0%B8%D0%BE%D0%BD%D0%BD%D0%BE%D0%B5_%D0%BA%D1%80%D0 %B0%D1%81%D0%BD%D0%BE%D0%B5_%D1%81%D0%BC%D0%B5%D1% 89%D0%B5%D0%BD%D0%B8%D0%B5) (и вообще-то теорией относительности и предсказано)?

iLLi-DAN
27.06.2010, 15:54
Эйнштейн пошёл намного дальше своих предшественников и показал, что ключевым следствием из гравитационного красного смещения является гравитационное замедление времени

Эйнштейн впервые предположил, что потерю энергии фотоном при переходе в область с более высоким гравитационным потенциалом можно объяснить через разность хода времени в точках приёма и передачи сигнала.


Таким образом, если время для приёмника и передатчика течёт с разной скоростью, наблюдаемая частота излучения, а вместе с ней и энергия отдельных квантов, тоже будет различной для приёмника и передатчика.

Тоесть мы (находясь в точке приема) видим замедленный свет?

Dragon27
27.06.2010, 15:59
Тоесть мы (находясь в точке приема) видим замедленный свет?
Свет с пониженной частотой. Локальная скорость света всегда и везде одна и та же

iLLi-DAN
27.06.2010, 16:02
Свет с пониженной частотой. Локальная скорость света всегда и везде одна и та же

А отчего понижается частота света?

Dragon27
27.06.2010, 16:16
А отчего понижается частота света?
Как бы так сказать. Частота света не понижается. Она уже разная в разных системах отсчёта из-за разного темпа хода времени. Гравполе-то статическое.
http://ufn.ru/ru/articles/1999/10/d/

iLLi-DAN
27.06.2010, 16:30
Как бы так сказать. Частота света не понижается.
http://ufn.ru/ru/articles/1999/10/d/

Так-так вот мы и пришли к тому что я рассказывал в первом посте, демонстрируя на картинке

моя цитата

Теория относительности же утверждает что длинна волны увеличивается потому время для приемника №2 и источника света (земля) время течет с разной скоростью, ЧТО МЫ ВИДИМ ЗАМЕДЛЕННЫЙ СВЕТ №4, а эйнштейн не подумал что мы будучи в точке №2 видим свет не тот который сейчас замедленный, а тот который до нас дошел №5 – НЕЗАМЕДЛЕННЫЙ. По сути никакого красного смещения быть не должно, А ОНО ЕСТЬ

Dragon27
27.06.2010, 16:33
ЧТО МЫ ВИДИМ ЗАМЕДЛЕННЫЙ СВЕТ
Неверно. Попросту в разных системах отсчёта свет имеет разные частоты. Красного смещения не может не быть.
Прочитай ещё раз статью в УФН внимательней. Там всё до мелочей объясняется.

iLLi-DAN
27.06.2010, 17:06
Хорошо, разберем то, что описывает теория относительности.

http://s53.radikal.ru/i140/1006/fd/7598382e386c.jpg (http://www.radikal.ru)

Время в гравитационном поле земли идет медленнее, и по отношению к приемнику в точке 3 частота света будет меньше, (НО КАК время влияет на частоту), в соответствии с ТО ( свет в точке 2 распространяется медленнее, так как время там течет медленнее) будете отрицать?
И частота света в точке 2 будет другая только по отношению к точке 3

Так что фотон краснеет только относительно часов. Часов в точке 3

Dragon27
Как бы так сказать. Частота света не понижается.
Попросту в разных системах отсчёта свет имеет разные частоты.

Dragon27
27.06.2010, 17:18
НО КАК время влияет на частоту
Источник света 2 тикает медленнее и частота исходящего от него сигнала медленнее.

свет в точке 2 распространяется медленнее, так как время там течет медленнее
Относительно источника 2 свет не меняет свою скорость. Локальная скорость света не меняется.

И частота света в точке 2 будет другая только по отношению к точке 3
С точки зрения источника 2 частота света одна. С точки зрения приёмника 3 частота света будет меньше. Соответственно, приёмник 3 будет воспринимать свет с пониженной частотой (красное смещение).
Всё верно?

iLLi-DAN
27.06.2010, 17:45
И


С точки зрения источника 2 частота света одна. С точки зрения приёмника 3 частота света будет меньше. Соответственно, источник 3 будет воспринимать свет с пониженной частотой (красное смещение).
Всё верно?

Отлично это то что я хотел услышать и понял это я уже очень давно.
А теперь вдумаемся, почему источник 3 будет воспринимать свет с пониженной частотой

Представим себе как свет испущенный с земли летит к приемнику, на земле произошла вспышка света, и (фотоны, кванты или что там этот свет) полетели к приемнику, вот они летят в грави поле земли, где время печет медленнее чем на приемнике, на приемнике света еще не видно, вот свет преодолел уже половину грави поля земли и замедление времени начало ослабевать( представим что мы летим со светом), вот наш свет долетел до края грави поля земли где замедления времени уже почти нет(приемник нас еще не видит) и вот мы вместе со светом вылетели из гр поля земли и оно больше не воздействует на свет, не замедляет его ( частота света осталась та же) приемник этот свет еще не увидел, свет дошедший до приемника не должен менять свою чатоту.

Проведи мысленный експеримент, педставь и источник и приемник не поотдельности, а вместе, все происходящее в реальном времени.

Dragon27
27.06.2010, 17:52
вот они летят в грави поле земли, где время печет медленнее чем на приемнике, на приемнике света еще не видно, вот свет преодолел уже половину грави поля земли и замедление времени начало ослабевать( представим что мы летим со светом), вот наш свет долетел до края грави поля земли где замедления времени уже почти нет(приемник нас еще не видит) и вот мы вместе со светом вылетели из гр поля земли и оно больше не воздействует на свет, не замедляет его
Ещё разок прочитайте, пожалуйста статью Л. Б. Окуня (хотя бы в первый раз :) ). Там чётко написано: свет не замедляется в гравитационном поле, частота света не меняется в фиксированной инерциальной системе отсчёта, пока свет летит в статическом гравитационном поле. С точки зрения приёмника источник испускает свет с уже пониженной частотой, и свет, не изменяясь, с этой пониженной частотой долетает до приёмника.

iLLi-DAN
27.06.2010, 18:12
С точки зрения приёмника источник испускает свет с уже пониженной частотой, и свет, не изменяясь, с этой пониженной частотой долетает до приёмника.

ТАК точему же частота понижается
значит мощность гравитационного поля не влияет на замедление времени, и то что по мере полета света в грави поле земли которое постоянно ослабевает замедление времени тоже не ослабевает.

Dragon27
27.06.2010, 18:16
замедление времени тоже не ослабевает
Пролёт света в статическом гравполе не влияет на сам свет в фиксированной инерциальной системе отсчёта.

ТАК точему же частота понижается
Частота света более низкая из-за смены системы отсчёта. Источник выпускает свет определённой частоты. С точки зрения приёмника источник света выпускает свет более низкой частоты, чем источник сам у себя фиксирует.
Прочитай статью.

iLLi-DAN
27.06.2010, 19:23
Ну наконец-то

Dragon27
Пролёт света в статическом гравполе не влияет на сам свет в фиксированной инерциальной системе отсчёта. (на сам свет как таков, на само явление)

А оказывается что физическое явление изменяется просто потому что мы поменяли систему отчета!!!

Dragon27
Частота света более низкая из-за смены системы отсчёта.

Есть задача Зенона
Быстроногий Ахиллес никогда не догонит черепаху, если в начале движения черепаха находится впереди на некотором расстоянии от него.
Допустим, Ахиллес бежит в десять раз быстрее, чем черепаха, и находится от неё на расстоянии в 1 километр. За то время, за которое Ахиллес пробежит этот километр, черепаха проползёт 100 метров. Когда Ахиллес пробежит 100 метров, черепаха проползёт ещё 10 метров, и так далее. Процесс будет продолжаться до бесконечности, Ахиллес так никогда и не догонит черепаху.
А почему Ахил не догонит черепаху, да потому что каждый отрезок расстояния изменяется система отчета.

Dragon27
Частота света более низкая из-за смены системы отсчёта.

И статью я прочитал сразу.

А теперь про то как же свет изменяет свою частоту.
Допустим на земле 1 и в космосе 2 на одинаковом расстоянии от приемника (за пределами гравии поля земли) произошли две вспышки света. Свет испущенный из космоса который идет к приемнику, не испытывает никаких изменений и прошел за 2 секунды 600 тыс.км. А свет испущенный с земли ( на которой время течет медленнее только для приемника но не для самой земли) прошел допустим 598 тыс.км в системе координат приемника и второй вспышки, но для земли он прошел 600 тыс.км И по отношению к свету испущенному из космоса его частота будет казаться меньше но только по отношению к свету испущенному в космосе и приемнику.

Dragon27
С точки зрения приёмника источник света выпускает свет более низкой частоты, чем источник сам у себя фиксирует.

Да, так и получается, но как то что мы измеряли частоту в другой системе отсчета может повлиять на физические свойства света
Так что фотон краснеет только относительно часов но мы же видим красный свет (потерявший энергию)

Предлагаю мою задачку –
С земли выстрелили очередь из пулемета по приемнику (находящемуся вне гравиполя земли) интервал между выстрелами 1секунда, скорость пули 1000 км/сек, и если пренебречь земным притяжением и трением воздуха (ради эксперимента), а учитывать только замедление времени , то с какой частотой пули ударят по приемнику, изначальной или нет???

Dragon27
27.06.2010, 19:46
А оказывается что физическое явление изменяется просто потому что мы поменяли систему отчета!!!
Частота света меняется от того, что мы меняем систему отсчёта. Это для тебя открытие?

(на сам свет как таков, на само явление)
Не забывай про систему отсчёта. Если мы, например, будет рассматривать летящий фотон с точки зрения инерциальных систем отсчёта локально-сопутствующих атомам, расположенных на уровнях гравпотенциала по пути летящего фотона, то энергия фотона будет уменьшаться, так как мы меняем ИСО. Если мы рассмотрим из одной фиксированной инерциальной системы отсчёта, то фотон частоту менять не будет.

А почему Ахил не догонит черепаху, да потому что каждый отрезок расстояния изменяется система отчета.
То есть, по-вашему, Ахилл не догонит черепаху? И при чём тут системы отсчёта?

но как то что мы измеряли частоту в другой системе отсчета может повлиять на физические свойства света
Там же чёрным по белому написано:
энергетические уровни у поглощающего атома чуть "синее", чем у испускающего атома.

Предлагаю мою задачку –
С земли выстрелили очередь из пулемета по приемнику (находящемуся вне гравиполя земли) интервал между выстрелами 1секунда, скорость пули 1000 км/сек, и если пренебречь земным притяжением и трением воздуха (ради эксперимента), а учитывать только замедление времени , то с какой частотой пули ударят по приемнику, изначальной или нет???
Если в СО пулемёта частота 1выс/сек, то в СО приёмника частота, разумеется, будет иная.

Кстати, такой эффект мы можем заметить в чёрной дыре. Подлетающий наблюдатель будет уверен, что у него время течёт нормально. Внешний наблюдатель будет чётко видеть, что все процессы подлетающего наблюдателя замедлены. И свет от него тускнеет.

iLLi-DAN
27.06.2010, 20:29
Если в СО пулемёта частота 1выс/сек, то в СО приёмника частота, разумеется, будет иная.


А я споросил не какая частота будет в СО приемника А скакой частотой пули реально ударили бы по нему?

И вообще я уже устал, кроме того тема называется

Теория: Ретрансляторы.
а не
Теория:Относительности.

Dragon27
27.06.2010, 21:03
А я споросил не какая частота будет в СО приемника А скакой частотой пули реально ударили бы по нему?
А с какой частотой волновые максимумы ударили бы по приёмнику? Частота воспринимаемая приёмником будет не равна частоте испускаемой источником. Грав. замедление времени влияет на все периодические процессы.

luden
28.06.2010, 12:11
И почему люди судят о книге по первому абзацу, не утруждая себя почитать хоть пол дня, и называют мукулатурой потому что автор назвал ее самостоятельной ветвью физики.
Я называю это мукулатурой, потому что это популизм. Вместо того, чтобы писать научные статьи и отстаивать свою точку зрения в научном сообществе, человек пишет для широкой публики, которая априори ничего этому движению дать не может. Это значит, что класть он хотел на науку, а занимается банальным срубанием бабла. Для того, чтобы различить мошенника иногда не нужно даже его труды читать, достаточно обычных поведенческих признаков, такие дела.
По крайней мере он пытается дать конструктивные ответы на вопросы, на которые современная физика отвечает просто НЕ ЗНАЮ.
Это потому что наука, в отличии от псевдонауки не умеет врать. И любой человек с наукой связанный, когда ему рассказывают про новое ответвление, которое точно знает как устроен мир понимает, что перед ним либо одураченный, либо лжец.
после горизонта событий в ЧД, начинается пространство с мнимой для нас кривизной.
Мнимая кривизна требует ещё одного, не описанного, измерения. А оно в свою очередь не может существовать только в чёрной дыре, измерение же. Вообще ты какую теорию рассматриваешь? Просто с точки зрения классической физики, всё очевидно, ЧД это просто очень тяжёлая фигня. Для ОТО я глуповат, но думается там должно получится, нечто вроде пространственного кармана.

Dragon27
28.06.2010, 19:32
http://www.astronet.ru/db/msg/1180462/node2.html
Если кому интересно чуть получше понять физику ЧД.

Zosia65
29.06.2010, 11:24
luden, В кванте ЭМ излучения, электрическое и магнитное поле, существуют не одновременно, а по очереди. Между ними есть точки, в которых вообще все отсутствует. Где находится энергия в такие моменты? Хотите, не хотите, а еще измерения существуют. И не только в ЧД. Мнимое искривление требует еще измерения? Можно подумать обычное не требует. А вот по поводу пространственного кармана... Я об этом и говорю. Для нас сфера, для нее расширяющееся пространство.

Dragon27, в примере с гравитационным замедлением времени, совершенно не учитывается факт гравитационного уменьшения протяженности пространства. Нету там красного смещения.

В зоне влияния ЧД, обычные формулы относительности не корректны. Тело находится в не инерциальной системе отсчета, ОНО ИСПЫТЫВАЕТ УСКОРЕНИЕ. В горизонте событий, пространство по отношению к внешнему имеет скорость (искривлено) света. А тело оказавшееся в нем не может достичь такой скорости по определению. Так оно что, имеет в нем отрицательную скорость ? Какой смысл формулы Е=МС^2, если тело будет бесконечно падая бесконечно же наращивать свою энергию...

Гравитация искривляет пространство. Бред. Искривление пространства, это и есть гравитация.

luden
29.06.2010, 11:39
Между ними есть точки, в которых вообще все отсутствует.
А вот это, батенька, нужно ещё доказать.
Хотите, не хотите, а еще измерения существуют.
Я в общем-то не против. Просто не стоит их клепать по мере того, что у нас что-то куда-то не влазит.
Для нас сфера, для нее расширяющееся пространство.
М-м-м-м, а центр масс в такой системе будет не в центре кармана? Раз уж он является образующей.

Zosia65
29.06.2010, 13:29
Лан, клепать не будем. Искривление происходит по временному измерению.
Теперь о точках. Рассмотрим другие моменты. Вот магнитное поле появилось. Начался рост, а где этот рост сидел? Проходим максимум, и начинаем куда то деваться... Задевались. Не. В следующей полу волне мы материализуемся неизвестным способом уже электрическим полем. Ну они порождают друг друга, ведь так же? Только как то замысловато,- появилось, где то зачало и ушло. Следствие появляется ПОСЛЕ ИСЧЕЗНОВЕНИЯ причины. Легко объяснимо при принятии квантовой природы пространства-времени. Квант, не является нулем (именно невозможность нулевого состояния времени, ставит ограничения на скорость распространения света). Следующим квантом будет состояние в котором отличия наступили. Ну и понятно неравновесные состояния кванта и инициируют последующие его изменения.

Центр масс будет в центре для внешнего наблюдателя. Да и лежит он под поверхностью горизонта событий. Согласитесь, для падающего все выглядит иначе. Падающее перед ним, улетает от него с ускорением, он в свою очередь падает с ускорением удаляясь от последующего. Да от меня все разлетается, может заявить он, навен это большой взрыв был... Похоже на наши представления? Случайно ли это?

luden
29.06.2010, 15:52
Начался рост, а где этот рост сидел? Проходим максимум, и начинаем куда то деваться... Задевались.
В момент падения одной волны мы имеем рост другой.
Да от меня все разлетается, может заявить он, навен это большой взрыв был...
Не может. Взрыв или падение, это вопрос направления, а не разницы скоростей.

Dragon27
29.06.2010, 18:59
luden, В кванте ЭМ излучения, электрическое и магнитное поле, существуют не одновременно, а по очереди. Между ними есть точки, в которых вообще все отсутствует.
Электромагнитное поле (единая сущность, не стоит её кромсать на отдельные подполя) есть везде. А там, где напряжённость равна нулю, попросту напряжённость равна нулю. Как тихая гладь моря. Если её взбеленить, по ней побегут волны, как электромагнитные волны по электромагнитному полю.

Где находится энергия в такие моменты?
В других местах (соседних точках). Только не говорите, что энергия - это независимая сущность. Энергия - это свойство сущностей. Это способность совершать работу.

Dragon27, в примере с гравитационным замедлением времени, совершенно не учитывается факт гравитационного уменьшения протяженности пространства. Нету там красного смещения.
Шо ещё за гравитационное уменьшение протяжённости пространства, новый термин? В статгравполе протяжённость пространства не меняется. Это же не космологическое расширение.
А красное смещение - вот оно. Теоретически предсказано, на опыте обнаружено.

В зоне влияния ЧД, обычные формулы относительности не корректны.
Обычные формулы специальной теории относительности. А не общей.

А тело оказавшееся в нем не может достичь такой скорости по определению.
Враньё.

Какой смысл формулы Е=МС^2, если тело будет бесконечно падая бесконечно же наращивать свою энергию...
Где ж оно будет бесконечно падать?

Гравитация искривляет пространство. Бред. Искривление пространства, это и есть гравитация.
Верно, только не просто пространство, а пространство-время. А искривляет его, разумеется, материя.

iLLi-DAN
29.06.2010, 20:28
Dragon27

А теперь про то как же свет изменяет свою частоту в соответствии с т.о -
Допустим на земле 1 и в космосе 2 на одинаковом расстоянии от приемника (за пределами гравии поля земли) произошли две вспышки света. Свет испущенный из космоса который идет к приемнику, не испытывает никаких изменений и прошел за 2 секунды 600 тыс.км. А свет испущенный с земли ( на которой время течет медленнее только для приемника но не для самой земли) прошел допустим 598 тыс.км в системе координат приемника и второй вспышки, но для земли он прошел 600 тыс.км И по отношению к свету испущенному из космоса его частота будет казаться меньше но только по отношению к свету испущенному в космосе и приемнику.(МЫ ИЗМЕРЯЕМ ЗАМЕДЛЕННЫЙ ЗЕМНОЙ СВЕТ) Мы измеряем расстояние которое прошел свет на земле по времени приемника, в результате земной свет сделал меньшее количество колебаний( по времени приемника) ТАК?

Dragon27
29.06.2010, 21:19
То есть, 1 на земле, а 2 и приёмник далеко в космосе на одинаковом расстоянии от Земли?
Свет от 2 почти не испытывает влияние тяготения и доходит до приёмника как обычно. Свет с земли испытывает красное смещение и гравитационную задержку сигнала (эффект Шапиро). То есть свет с земли пролетит меньшее расстояние.
А количество колебаний, что конкретно ты под этим имеешь в виду? Частота для приёмника будет ниже, и колебаний для него будет меньше, если ты это имеешь в виду.

iLLi-DAN
29.06.2010, 22:44
Dragon27
Так значит замедленный свет испущенный замедленным атомом, выйдя из гравиполя (в котором время замедленно) земли так и останется замедленным? и воспримется для приемника замедленным?

Еще один мысленный эксперимент, есть планета, на которой замедление времени составляет 50%, там живут люди, если мы посмотрим на них в телескоп с орбиты где замедления времени нету, мы увидим их на 50% замедленными?

Dragon27
29.06.2010, 22:58
Так значит замедленный свет испущенный замедленным атомом, выйдя из гравиполя (в котором время замедленно) земли так и останется замедленным? и воспримется для приемника замедленным?
Свет с определённой частотой (что за обозначение - замедленный свет?) пройдя через статическое гравитационное поле не будет менять свою частоту в ИСО приёмника. С чего бы?

Статью помнишь?

Добавлено через 1 минуту
Еще один мысленный эксперимент, есть планета, на которой замедление времени составляет 50%
На 50% замедлены в системе отсчёта телескопа с орбиты. Нет никаких на 50% замедлены вообще. Только в какой-то системе отсчёта.

iLLi-DAN
30.06.2010, 00:27
Свет с определённой частотой (что за обозначение - замедленный свет?) пройдя через статическое гравитационное поле не будет менять свою частоту в ИСО приёмника. С чего бы?

А в какой же момент меняется частота? В КАКОЙ момент?

А если частота не меняется
то из за чего атом испускает кванты реже, из за того что время течет на земле медленнее по отношению к приемнику ТАК?

На 50% замедлены в системе отсчёта телескопа с орбиты.
именно так
если мы посмотрим на них в телескоп с орбиты где замедления времени нету, мы увидим их на 50% замедленными?

Dragon27
30.06.2010, 01:06
А в какой же момент меняется частота? В КАКОЙ момент?
Частота задаётся в момент излучения, и с тех пор не меняется.

то из за чего атом испускает кванты реже, из за того что время течет на земле медленнее по отношению к приемнику ТАК?
Не только кванты реже, но и сами кванты имеют меньшую частоту.

именно так
если мы посмотрим на них в телескоп с орбиты где замедления времени нету, мы увидим их на 50% замедленными?
Не совсем точно помню формулы. Допустим.

ZeroCoolDark
30.06.2010, 07:48
Мой свернутый и спресованный, как центр ЧД мозг сказал мне, что тему стоит переименовать.

Zosia65
30.06.2010, 08:23
"факт гравитационного уменьшения протяженности пространства". Смотрим http://ru.wikipedia.org/wiki/Лоренцево_сокращение. Учитываем, что эффект вызван гравитацией ЧД. Стараемся понять собеседника, а не цепляемся к знакомым словам без контекста. Ну а особо продвинутые разъясняют нам нюансы вызванные не инерциальной СО возле гравитирующего тела в сравнении с внешним наблюдателем.
Следует помнить особенности нашего языка: при попытках описывать парадоксы времени, Вы неизбежно будете не точны. Смысловые конструкции в языке строятся без учета относительности и не приспособлены к отображению оного.

Dragon27
30.06.2010, 19:19
Смотрим http://ru.wikipedia.org/wiki/Лоренцево_сокращение.
И где тут гравитационные эффекты? Лоренцево сокращение - это чисто кинематический эффект СТО, а не метрический эффект ОТО. Хотя и у ОТО есть свои хитрые эффекты.

У темы страшное название…

Zosia65
01.07.2010, 11:50
Даем начать падание камню. Через некоторое время, начинаем падать сами. Через такое же время начинает падение камень вслед. Теперь смотрим. Камень падающий перед нами, постоянно наращивает скорость относительно нас. Мы сами, так же наращиваем скорость по сравнению с отстающим. С нашей точки зрения, камни разлетаются с ускорением. Расстояния растут со временем. Расширяющееся пространство-время.
Dragon27, скажите, почему по такому принципу нельзя рассматривать большой взрыв образующий нашу вселенную? В чем неприменимость такого подхода? В отсутствии такого взгляда в современной науке? Может уже существуют знания делающие такое рассмотрение заведомо ложным?

luden
01.07.2010, 12:21
Расстояния растут со временем.
А теперь попробуй смоделировать не разницу скоростей на прямой, а именно трёх-мерное. То есть, чтобы запороть эксперимент вместе с тобой в пяти метрах рядом прыгнул я. Камни от нас удаляются, а мы друг от друга ессно нет. Расширение уничтожено, осталось только то что есть, перемещающиеся объекты.

Dragon27
01.07.2010, 19:33
Даем начать падание камню. Через некоторое время, начинаем падать сами. Через такое же время начинает падение камень вслед. Теперь смотрим. Камень падающий перед нами, постоянно наращивает скорость относительно нас. Мы сами, так же наращиваем скорость по сравнению с отстающим. С нашей точки зрения, камни разлетаются с ускорением. Расстояния растут со временем. Расширяющееся пространство-время.
Dragon27, скажите, почему по такому принципу нельзя рассматривать большой взрыв образующий нашу вселенную? В чем неприменимость такого подхода? В отсутствии такого взгляда в современной науке? Может уже существуют знания делающие такое рассмотрение заведомо ложным?
А в чём собственно вопрос? То, что в нашей Вселенной любой объект считает себя центром, даже если этого центра не существует - это и так ясно.

Zosia65
02.07.2010, 11:30
luden, ну Луна, да и Солнце, от нас не удаляются, не смотря на расширение Вселенной. О чем это говорит? Мы не расширяемся? Ну станете падать в пяти метрах от меня, с каждым пройденным километром эти пять метров будут уменьшаться. И это не из за взаимного притяжения, одна геометрия искривленного пространства.
Для около светового расширения, мы должны падать вблизи горизонта событий, или за ним. Рассматривать в трех измерениях? Вы понимаете, что СО в гравитационном искривлении пространства становится не инерциальной? Там характеристики времени-пространства не являются линейными величинами. Это для тел разлетающихся (сближающихся) со скоростями света уменьшается ТОЛЬКО длина (протяженность по вектору движения) относительно друг друга. Условия ЧД, меняют в том числе измерения ширины. Сами упоминали решения в комплексных плоскостях. Вне ЧД, эффектами релятивистских изменений ширины можно смело пренебрегать. Вблизи, будьте любезны учитывать кривизну пространства.
Dragon27, в чем принципиальная разница падающего с камнями меня, и нашей Вселенной? Почему я не могу поставить знак равенства своего падения в ЧД, и большого взрыва Вселенной? Потому, что этакого Вы нигде не читали? Мы будем падать с luden параллельными курсами, и обязательно пересеклись бы... В точке сингулярности...

luden
02.07.2010, 11:37
Zosia65, ты уж определись, что ты хочешь доказать. Началось всё с того, что ты считаешь, что у ЧД нет явно выраженного ядра. Далее последовал пример с разлетающимися камнями. Теперь мы сближаемся, а камни разлетаются. И тд.

V_Nick
02.07.2010, 15:53
Эх, стать бы сейчас какой-нибудь мега-субстанцией с возможностью перемещения со скоростью света, посмотреть как что устроено и рассказать ученым.
А то все спорят о гипотетических объектах, которых, может даже, на самом деле, не существует.
А Теория Относительности - научная религия, которой поклоняются и презирают неверных.
Любая другая теория, объясняющая процессы во вселенной, кроме ТО имеет место быть, и не должна высмеиваться и вытесняться энштейнистами за бессмысленные допущения. Как будто у самого Энштейна нет никаких тупых допущений. Взять хотя бы большой взрыв - "Допустим была сингулярность, допустим она взорвалась". А откуда она взялась, почему она взорвалась, какова причина взрыва (закон причинности никто не отменял), и почему это произошло именно около 13 млрд лет назад, видимо только одному Энштейну и его сторонникам понятно, типа коммерческий секрет. И как всегда ответ у сторонников теории Энштейна всегда туманный - "мы этого пока не знаем".
А как же тогда это узнать? Формулу придумать? Формулы и выкладки - это математический "язык", с помощью которого как и на обычном разговорном языке, можно "обманывать" и изворачиваться, делать нелепые допущения.
Теория Относительности недалеко ушла от Астрологии, одна лишь разница в том, что там все официально в цифрах, все уравнения, с допущениями, сходятся, нобель уже получен и т.д.

Dragon27
02.07.2010, 19:19
Dragon27, в чем принципиальная разница падающего с камнями меня, и нашей Вселенной?
В размерах :)
Ещё раз, что вы хотите объяснить? Разнообразные теории о том, что мы живём в ЧД и прочее - это просто трёп. Но даже этот трёп нужно уметь грамотно излагать, без всяких "мнимых" кривизн и прочего. Изучите вы ОТО по нормальной литературе, а не научной попсе. А так просто у вас нести идеи не получится.

Любая другая теория, объясняющая процессы во вселенной, кроме ТО имеет место быть, и не должна высмеиваться и вытесняться энштейнистами за бессмысленные допущения.
Если кто-то создаст нормальную теорию, никто его высмеивать не будет (из учёных). СТО - это практически Ньютонова механика в своё время. Она слишком хорошо подтверждена, чтобы её считать религией. И то всякое (http://arxiv.org/abs/hep-ph/0601236) выдумывают. А на ОТО существует кучи (http://en.wikipedia.org/wiki/Alternatives_to_general_relativity) альтернативных теорий гравитаций. ОТО всего лишь самая простая из этих теорий, а для многих теорий является предельным упрощением.

"Допустим была сингулярность, допустим она взорвалась"
Чушь, теория динамической расширяющейся вселенной формировалась совершенно другим историческим путём. Эйнштейн, между прочим, специальный член в формулы вводил, чтобы получилась стационарная Вселенная (тогда ещё про другие галактики не знали, какой там про космологическое красное смещение).

почему она взорвалась, какова причина взрыва (закон причинности никто не отменял), и почему это произошло именно около 13 млрд лет назад
Вы можете прочитать в многочисленной литературе (Вайнберг, Зельдович, Постнов, прочие) по теории классического БВ и инфляции.

Формулы и выкладки - это математический "язык", с помощью которого как и на обычном разговорном языке, можно "обманывать" и изворачиваться, делать нелепые допущения.
Грамотный человек способен распознать нормальную речь и отличить от демагогии. Грамотный физик способен отличить математические фокусы от физических формул с физическим смыслом. И не на одних формулах ТБВ держится, ещё и на наблюдениях и даже кое-каких предсказаниях.

luden
04.07.2010, 09:30
Теория Относительности недалеко ушла от Астрологии, одна лишь разница в том, что там все официально в цифрах, все уравнения, с допущениями, сходятся, нобель уже получен и т.д.
А ещё она работает. У тебя критика как у бабки, - понапридумывают всякого, фулюганы, а им ишшо и деньги плотют.

Zosia65
04.07.2010, 13:39
Dragon27, В моем примере падения в ЧД с камнями, подскажите пожалуйста мне физический эксперимент, для установления факта падения в оную, а не нахождения в расширяющейся вселенной. Мне не с чем сравнить размеры :(. Уже давно упавшее до меня и падающее за мной, имеют скорости сопоставимые со скоростью света.

Dragon27
04.07.2010, 14:44
Dragon27, В моем примере падения в ЧД с камнями, подскажите пожалуйста мне физический эксперимент, для установления факта падения в оную, а не нахождения в расширяющейся вселенной.
Зачем? Вам нужны факты для различения НИСО и ИСО с фиктивным гравитационным полем? Мы не вращаемся вокруг Солнца, это хитрое гравитационное поле всё вокруг вертит, а Земля в центре Вселенной :)

У классической модели горячей Вселенной есть некоторые проблемы. Эти проблемы успешно и относительно просто решает модель инфляционной Вселенной с очень хитрыми темпами расширения. Каким образом они соотносятся с вашей гипотетической 4D ЧД?

Zosia65
05.07.2010, 09:49
Dragon27, жду опыт. Хватит словоблудить.

V_Nick
05.07.2010, 10:02
Вы можете прочитать в многочисленной литературе (Вайнберг, Зельдович, Постнов, прочие) по теории классического БВ и инфляции.

Я мог бы многое почитать и осмыслить, только делать этого нихочу, иначе начну думать также как в книге. Мне гораздо ближе теории, которые лучше поддаются здравому смыслу и логике. Например теории эфирного пространства.

Грамотный человек способен распознать нормальную речь и отличить от демагогии. Грамотный физик способен отличить математические фокусы от физических формул с физическим смыслом. И не на одних формулах ТБВ держится, ещё и на наблюдениях и даже кое-каких предсказаниях.
Ага, верим, верим. Грамотный человек способен знать, что черные дыры существуют. Грамотный человек верит формулам что был когда-то большой взыв (непонятно по каким причинам, но все же был) и все вокруг произошло из ничего. А глупцы не верят и что-то свое придумывают. Вот так и везде у нас.
Ведь черный квадрат Малевича, просто, тупо, черный квадрат, но при этом ведь какое высокое искусство! Да Вы что, это же Малевич!

А ещё она работает. У тебя критика как у бабки, - понапридумывают всякого, фулюганы, а им ишшо и деньги плотют.

А у тебя знания как у верующего. "Да здравствует Энштейн! Да придет с нами сила Большого Взрыва! Да воплотятся же формулы, которые в теории работают, но на практике невозможно доказать".

luden
05.07.2010, 10:17
А у тебя знания как у верующего. "Да здравствует Энштейн! Да придет с нами сила Большого Взрыва! Да воплотятся же формулы, которые в теории работают, но на практике невозможно доказать".
Лолшто? Формулы уже сто лет в обед подтверждены экспериментально. То что тебе где-то на левых говносайтах промыли мозги, это конечно печально, но давай не будем на этой основе действующие теории под сомнения ставить. У тебя есть конкретная критика СТО? ОТО? ТБВ? Нет же. У тебя из всей критики - ничё не понял, мне кажется что моя версия логичнее. Может у тебя есть теория, которая лучше объяснеят ситуация. Но только теория, а не заявление, что вот так вот лучше. Какие факты говорят в твою пользу и тд.
Ты хоть знаешь на чём основывается ТБВ? Что это не на пустом месте выдумка, какие факты говорят в пользу его.

Если уж говорить о моих знаниях. То знаю я одно, любая научная теория неверна в общем. Можно говорить о каких-то рамках. Так вот в своих рамках теории Эйнштейна верны.

Zosia65
05.07.2010, 10:24
V_Nick, наверно это следствие мятежности духа. Флаг тебе в руки. Ортодоксы никогда не двигали науку. Эфирное пространство... Скорее всего под эфиром выступает гравитационное поле. Оно есть везде. Там где его нет, пространство становится абстрактной математической функцией.

luden
05.07.2010, 10:36
Zosia65, нэ, гравитация здесь не причём. Эфир это теоретическая субстанция, колебания которой это и излучения, и эм волны и вообще всё на свете. Опять же в своих рамках теория применима и в чём-то определённо удобна. Но теория заглохла и дальнейшего развития не получила.
А уровнять эфир и гравитационные поля ещё Эйнштейн предлагал.
В общем в будущем, если теории Эйнштейна зайдут в тупик может и будет смысл воскресить эфирные, но сейчас не целесообразно.

Zosia65
05.07.2010, 10:47
luden, гравиполе колебаться не может? Постоянство скорости света может вытекать именно из его свойств. И горизонт событий у ЧД, тоже возможно следствие его свойств. Искривление пространства, уже принятое его свойство. Да другого претендента на звание эфира пока (пока ли...) просто не существует.

luden
05.07.2010, 10:53
Zosia65, это вот ты сейчас предложение Эйнштейна озвучиваешь, загнать эфир в рамки ОТО. А товарищ вот видишь его антихристом считает. А эфирные теории в оригинале, это как раз среда распространения эм волн и пр. То есть сама по себе.

V_Nick
05.07.2010, 15:27
Формулы уже сто лет в обед подтверждены экспериментально. То что тебе где-то на левых говносайтах промыли мозги, это конечно печально, но давай не будем на этой основе действующие теории под сомнения ставить.

Ну, мозги никто мне не промывал, это как раз у тебя мозг промыт основательно.
Хорошо и удобно оперировать теориями, которые практически доказать нельзя, а теоретически не опровергнуть. С таким успехом можно выдвинуть теорию об "Алисе в стране чудес", математически описать все процессы происходящие в зазеркалье. Хороша будет теория, из разряда "хотите верьте, хотите нет". То же самое про черные дыры и большой взрыв. Теория имеется, все математически доказано, опровержений нет, доказательств тоже не будет никогда. Остается только верить, как в бога. Или не верить, но будешь высмеян за неграмотность и презрен сторонниками данной теории.

У тебя есть конкретная критика СТО? ОТО? ТБВ?

Тебе наверное будет смешно, но эти сокращения СТО, ОТО, ТБВ - мне ни о чем не говорят. Я уж не ботан по науке, так что просьба расшифровывать в дальнейшем.

У тебя из всей критики - ничё не понял, мне кажется что моя версия логичнее.
Так получилось, что астрономией увлекаюсь еще с юности, но читая сегодняшние получудесные теории, в глубине мозга бродит какая-то левая мысль, что меня где-то наё.. но не могу понять где. А версия эфирного пространства мне все таки ближе, так как более логичнее большого взрыва. И мне плевать, что я не докажу это формулами, это мое сугубо личное мнение.

То знаю я одно, любая научная теория неверна в общем. Можно говорить о каких-то рамках. Так вот в своих рамках теории Эйнштейна верны.
Так почему бы не направить знания на изучение, реальных, досягаемых вещей, а не компостировать мозг себе и окружающим всякими абстрактными теориями, которые дальше светлых головушек, да исписанных томов не выберутся никогда.

А товарищ вот видишь его антихристом считает
Это ты про кого? Если про меня, то я вовсе не считаю его таким. Он делал свое дело и был очень умен раз смог убедить научное сообщество и создать новое направление в науке. Ведь умные люди "глянец" не читают, умные люди его издают. :)

luden
05.07.2010, 15:42
Тебе наверное будет смешно, но эти сокращения СТО, ОТО, ТБВ - мне ни о чем не говорят. Я уж не ботан по науке, так что просьба расшифровывать в дальнейшем.
А, ну ок тогда. Прости, что что-то от тебя требую.

V_Nick
05.07.2010, 16:29
А, ну ок тогда. Прости, что что-то от тебя требую.

Это сарказм?

luden
05.07.2010, 16:33
V_Nick, скажем так, мне это действительно показалось смешным.

V_Nick
05.07.2010, 16:55
мне это действительно показалось смешным.
Высокомерность и насмешливость плохие качества. Они являются признаком внутренней неполноценности и мелкодушия.
Гордись этим, если больше нечем. :)

luden
05.07.2010, 17:08
V_Nick, гордиться чем, позволь спросить?

Dragon27
05.07.2010, 19:00
Dragon27, жду опыт. Хватит словоблудить.
Вы ещё что-то требуете? Подтверждений ваших фантазий от вас должны требовать, а не вы от нас. А словоблудия у вас, действительно, предостаточно.

Я мог бы многое почитать и осмыслить, только делать этого нихочу, иначе начну думать также как в книге.
Замечательно. "нихочу" ©
Попробуйте почитать, Д. С. Горбунов, В. А. Рубаков. Введение в теорию ранней Вселенной. 2-хтомник, хотя бы оглавление на сайте издательства (покупать уж я вас не заставляю).
Или вот (http://e-science.ru/forum/index.php?showtopic=10861), раздел "Космология".

Мне гораздо ближе теории, которые лучше поддаются здравому смыслу и логике. Например теории эфирного пространства.
Людям всегда были более близки дилетантские наукоподобные подражания, не выдерживающие строгого научного рассмотрения.

Грамотный человек способен знать, что черные дыры существуют. Грамотный человек верит формулам что был когда-то большой взыв (непонятно по каким причинам, но все же был) и все вокруг произошло из ничего. А глупцы не верят и что-то свое придумывают.
Грамотный человек способен оторваться от демагогии и заняться изучением серьёзной литературы (без которой понимание, увы, тщетно). Глупцы любят выдумывать всё, что в голову взбредёт, не изучая огромного опыта предыдущих поколений (нафига?).

В общем в будущем, если теории Эйнштейна зайдут в тупик может и будет смысл воскресить эфирные, но сейчас не целесообразно.
Полевой подход уже давно наглухо заткнул какие-то шансы эфира (как среды, колебания частиц которой создают излучение) на возрождение. Возродить можно только термин (эфир в химии, в СМИ).

Хорошо и удобно оперировать теориями, которые практически доказать нельзя, а теоретически не опровергнуть. С таким успехом можно выдвинуть теорию об "Алисе в стране чудес", математически описать все процессы происходящие в зазеркалье. Хороша будет теория, из разряда "хотите верьте, хотите нет". То же самое про черные дыры и большой взрыв. Теория имеется, все математически доказано, опровержений нет, доказательств тоже не будет никогда. Остается только верить, как в бога. Или не верить, но будешь высмеян за неграмотность и презрен сторонниками данной теории.
Ты всё-таки посмотри экспериментальный базис. Повторять фразы о недоказуемости можно хоть до конца жизни.

Так получилось, что астрономией увлекаюсь еще с юности, но читая сегодняшние получудесные теории, в глубине мозга бродит какая-то левая мысль, что меня где-то наё.. но не могу понять где.
Где-то недоучил (теории могут развиваться довольно шустро), отстал и подумал, что все врут.

И мне плевать, что я не докажу это формулами, это мое сугубо личное мнение.

Ну раз с научным мышлением покончили, тогда действительно спорить нечего.

Zosia65
06.07.2010, 10:09
Dragon27, глупо ждать от Вас описание опыта, показывающего неправомерность моего предположения. Впрочем, как и подтверждающего. Видимо сослаться не на что, а со своими решениями Вы осторожничаете. По меньшей мере. Понимаю, мое предположение ложно, потому что я дурак. А ведь действительно дурак, раз пытаюсь с Вами дискутировать...

V_Nick
06.07.2010, 10:15
Попробуйте почитать, Д. С. Горбунов, В. А. Рубаков. Введение в теорию ранней Вселенной. 2-хтомник, хотя бы оглавление на сайте издательства (покупать уж я вас не заставляю).
Или вот, раздел "Космология".

Какой толк от штудирования брошюрки? Ну вот ты штудировал, что тебе это дало? Что ты можешь доказать, сделаешь ли ты новое открытие в избранной тобой науке?
Нет. Ты лишь только поднатаскался в терминах и понятиях, чтобы выигрывать споры на форумах, у таких как я. А наука как стояла на месте, так и продолжает стоять. Люди по прежнему до конца не осознают даже таких вещей как собственный сон и строят догадки о внутреннем строении собственной планеты. Зато попридумывать разные фантастические теории это мы всегда рады.

luden
06.07.2010, 10:27
Zosia65, ты требуешь от него опыт по получению информации из-за горизонта событий? Are you seriously?
Ну вот ты штудировал, что тебе это дало?
Он как минимум представляет о чём говорит.

Zosia65
06.07.2010, 10:36
luden, неа. Вблизи, и снаружи. За горизонт в этом случае не лезу.

V_Nick
06.07.2010, 10:45
Он как минимум представляет о чём говорит.

Представлять это одно, а знать полезность этого, другое.

luden
06.07.2010, 10:48
неа. Вблизи, и снаружи. За горизонт в этом случае не лезу.
Наблюдатель будет видеть явно выраженное направление ускорения.
Представлять это одно, а знать полезность этого, другое.
Развлечение же.

Dragon27
06.07.2010, 21:15
глупо ждать от Вас описание опыта, показывающего неправомерность моего предположения.
Какой тут может быть опыт? Это всё равно что предложить проверить одну из тысяч вариаций теории струн современными способами (или хотя бы саму теорию струн).

Вот, например, моделирования Вселенной в ЧД:
http://arxiv.org/abs/1007.0587
http://arxiv.org/abs/0901.0215
Какие-то возможные проверки? Чистейшая теория.

Какой толк от штудирования брошюрки?
Это совсем не брошюрка :)

Ну вот ты штудировал, что тебе это дало? Что ты можешь доказать, сделаешь ли ты новое открытие в избранной тобой науке?
Вы от меня требуете сделать что-то новое в науке? Зачем? Вот я подучусь, и может пойду в какой-нибудь научный центр…

А что, кстати, можете сделать вы? :)

А наука как стояла на месте, так и продолжает стоять. Люди по прежнему до конца не осознают даже таких вещей как собственный сон и строят догадки о внутреннем строении собственной планеты.
До конца осознать подобные вещи, разумеется, сложно (особенно когда люди не понимают, что они хотят осознать, и пытаются придумать какие-то сверхметафизические сущности). А зачем строить догадки о внутреннем строении планеты, если можно опираться на экспериментальные данные и проверенные теории? Конечно, как любят говорить астрофизики, о внутреннем строении планеты мы знаем гораздо меньше, чем о внутреннем строении звёзд.
Наука от лженауки, вообще говоря, этим и отличается: она старается не врать. И если мы не можем точно описать условия в центре планеты, это не означает, что наука не права и надо брать на вооружение псевдодомыслы псевдоучёных.


Дискуссия, как всегда, пляшет, не задерживаясь на конкретных темах.

V_Nick
07.07.2010, 13:05
Вот я подучусь, и может пойду в какой-нибудь научный центр…

Ясно для чего, чтобы зарабатывать деньги. А чтобы зарабатывать деньги, кормить себя и прежде всего семью, необходимо не сидеть на месте взявшись за голову, а изображать постоянную занятость. А то за спиной будут шептаться, что работник некомпетентный попался. Проводить эксперименты, ставить опыты, полезности которых ноль, но в журнале галочка проставлена будет, а главное работа кипит.
Недосягаемость черных дыр, квазаров, да и других "интересных" объектов вселенной, только на руку. Вроде как главный козырь в науке. Можно придумывать и накручивать все что угодно, все равно ведь никогда не узнаем, как оно на самом деле.

Добавлено через 5 минут
А что, кстати, можете сделать вы?

В науке ничего. Я на это не учился.
Зато могу тебе компьютер оживить, конфиг 1с поправить, АТС перепрограммировать, с софтом разобраться и т.д. Я на это учился и этим зарабатываю, чтобы прокормить семью.

luden
07.07.2010, 13:38
Ясно для чего, чтобы зарабатывать деньги. А чтобы зарабатывать деньги, кормить себя и прежде всего семью, необходимо не сидеть на месте взявшись за голову, а изображать постоянную занятость. А то за спиной будут шептаться, что работник некомпетентный попался. Проводить эксперименты, ставить опыты, полезности которых ноль, но в журнале галочка проставлена будет, а главное работа кипит.
Недосягаемость черных дыр, квазаров, да и других "интересных" объектов вселенной, только на руку. Вроде как главный козырь в науке. Можно придумывать и накручивать все что угодно, все равно ведь никогда не узнаем, как оно на самом деле.
XD Всё невероятнее и невероятнее.
Зато могу тебе компьютер оживить, конфиг 1с поправить, АТС перепрограммировать, с софтом разобраться и т.д. Я на это учился и этим зарабатываю, чтобы прокормить семью.
Эникейщик, то бишь. Ну так, а чего о науке споришь?

V_Nick
07.07.2010, 13:49
Всё невероятнее и невероятнее.

К теориям о происхождении вселенной это тоже относится.

Вот например почитай это http://znaniya-sila.narod.ru/universe/uni000_06.htm

Ну так, а чего о науке споришь?
Развлечение же
Наука о вселенной мне интересна.

luden
07.07.2010, 17:04
К теориям о происхождении вселенной это тоже относится.
Да я не о том. Просто у тебя представления об учёном сообществе забавные.Развлечение же
Наука о вселенной мне интересна.
Но читать про неё ты не хочешь.

V_Nick
07.07.2010, 17:58
Просто у тебя представления об учёном сообществе забавные.

На своем веку уже много организаций повидал, со многими людьми общался, много выслушал от них историй, но в целом все везде схоже, а ключ к пониманию людской натуры - это то, что каждый прежде всего думает о себе и своей семье, а уж потом думает о заработке, и в конце концов о том, как деньги заработать. Энтузиазм как правило зациклен на себе. Это еще от обезьян осталось, стремление выглядеть во всем лучше другого и быть умнее всех.
Ради блага общества энтузиазмом никто не занимается, только ради себя любимого и ради заработка.

Добавлено через 1 минуту
Но читать про неё ты не хочешь.
Читать то я читаю, но только общие материалы, без формул и расчетов. Поверь, если я еще сейчас начну изучать формулы ОТО или СТО, то я наверное облысею и состарюсь за рекордно-короткий срок.

Dunkan
07.07.2010, 18:11
Меня название темы убивает)))

luden, это ты придумал?)

Dragon27
07.07.2010, 18:45
Ясно для чего, чтобы зарабатывать деньги.
Ну мне вообще-то очень нравится наука, и я надеюсь получать от неё хоть какое-то удовольствие (чисто от самого факта хотя бы :) ).

А чтобы зарабатывать деньги, кормить себя и прежде всего семью, необходимо не сидеть на месте взявшись за голову, а изображать постоянную занятость. А то за спиной будут шептаться, что работник некомпетентный попался. Проводить эксперименты, ставить опыты, полезности которых ноль, но в журнале галочка проставлена будет, а главное работа кипит.
Что ж поделать :)
Однако такие вещи в совокупности тоже приносят пользу. Хотя бы количеством. И простым изображением занятости занимаются всё-таки не особо компетентные работники.

Недосягаемость черных дыр, квазаров, да и других "интересных" объектов вселенной, только на руку. Вроде как главный козырь в науке. Можно придумывать и накручивать все что угодно, все равно ведь никогда не узнаем, как оно на самом деле.

Ну это же более теоретические темы, навряд ли я буду в них работать.

Зато могу тебе компьютер оживить, конфиг 1с поправить, АТС перепрограммировать, с софтом разобраться и т.д. Я на это учился и этим зарабатываю, чтобы прокормить семью.
А я на программиста учился (и учусь) и программистом работаю (базы данных программирую). И с подобными вещами сам постараюсь разобраться :)

Наука о вселенной мне интересна.
Вот только одного интереса маловато, чтобы разобраться в том, что успели разработать за сотню лет (начиная с СТО 1905 года). Нужно много времени тратить (хотя бы урывками) на чтение серьёзной, обучающей литературы, а не псевдонаучной попсы.

Читать то я читаю, но только общие материалы, без формул и расчетов. Поверь, если я еще сейчас начну изучать формулы ОТО или СТО, то я наверное облысею и состарюсь за рекордно-короткий срок.
А они не так сложны, как поначалу кажется. Жалко потратить пару-тройку лет для получения более-менее какого-то понимания? Людям обычно хочется всего и сразу.
А без математики, как ни крути, ни хрена понять не получится. А если понять хочется, придётся переломить своё к ней отношение.

V_Nick
08.07.2010, 17:02
Ну мне вообще-то очень нравится наука, и я надеюсь получать от неё хоть какое-то удовольствие
Это поначалу. Потом надоест одно и то же до мозга костей.

А я на программиста учился (и учусь) и программистом работаю (базы данных программирую). И с подобными вещами сам постараюсь разобраться
Да я вобщем-то и не навязывался. :)

А они не так сложны, как поначалу кажется. Жалко потратить пару-тройку лет для получения более-менее какого-то понимания? Людям обычно хочется всего и сразу.
А без математики, как ни крути, ни хрена понять не получится. А если понять хочется, придётся переломить своё к ней отношение.
Все это хорошо, когда есть для этого время.

Dragon27
08.07.2010, 19:19
Это поначалу. Потом надоест одно и то же до мозга костей.
Зависит от отношения. Помню один учёный говорил, что боится, что не успеет до конца жизни реализовать всё, что задумал. По этой причине он отказался от какой-то там должности (кучу времени потерять на ней).

Все это хорошо, когда есть для этого время.
Не можете урвать куски времени? Дело ваше, но для спора по теме необходимо хотя бы более-менее в ней разбираться.

V_Nick
09.07.2010, 11:58
Зависит от отношения.

Отношение со временем меняется у многих. Ты растешь и развиваешься, мозг тоже и мнение в 30 лет у тебя будет не такое как в 20. Сам увидишь.

Не можете урвать куски времени? Дело ваше, но для спора по теме необходимо хотя бы более-менее в ней разбираться.

Не могу. Отдыхать от всего тоже иногда охота. Я в споры конечно стараюсь не вступать и высказываю лишь свое мнение. Может оно и не правильное, но ведь глупо принимать чью-то точку зрения, когда не доверяешь ей или сомневаешься.

Dragon27
09.07.2010, 19:43
Отношение со временем меняется у многих. Ты растешь и развиваешься, мозг тоже и мнение в 30 лет у тебя будет не такое как в 20. Сам увидишь.
Поменяться-то поменяется, но я всё-таки надеюсь оставить любовь к науке (и к серьёзному глубокому научному пониманию. Это совсем не то, что любовь к научпопу, ощущения совсем иные), как и к программированию.

luden
12.07.2010, 12:16
а ключ к пониманию людской натуры - это то, что каждый прежде всего думает о себе и своей семье, а уж потом думает о заработке, и в конце концов о том, как деньги заработать.
Слишком примитивно. Да плюс к тому, учёный вполне способен зарабатывать нормальные деньги, а так как за айфонами и форд фокусами они редко спешат, то им вполне хватает того, что они получают в штатном режиме. Плюс зачастую мышление довольно искажено в сторону логики. Что, кстати, делает людей на удивление непредсказуемыми.
Читать то я читаю, но только общие материалы, без формул и расчетов. Поверь, если я еще сейчас начну изучать формулы ОТО или СТО, то я наверное облысею и состарюсь за рекордно-короткий срок.
Ну ОТО это наверное перебор, а СТО вполне себе можно.
luden, это ты придумал?)
Нет, это Зеро.
Отношение со временем меняется у многих. Ты растешь и развиваешься, мозг тоже и мнение в 30 лет у тебя будет не такое как в 20. Сам увидишь.
Наука вещь такая, чем больше углубляешься, тем невероятнее и интереснее результат.
Отдыхать от всего тоже иногда охота.
И всё таки. Развлечение же.

V_Nick
12.07.2010, 16:14
но я всё-таки надеюсь
Надежда умирает последней.. :) Я тоже в 22 года на многое надеялся.

Это совсем не то, что любовь к научпопу
Научпоп интересен именно тем, кто не зубрил ночами ОТО, СТО и прочие науки. Я среди них.

Слишком примитивно. Да плюс к тому, учёный вполне способен зарабатывать нормальные деньги, а так как за айфонами и форд фокусами они редко спешат, то им вполне хватает того, что они получают в штатном режиме. Плюс зачастую мышление довольно искажено в сторону логики. Что, кстати, делает людей на удивление непредсказуемыми.
Это конечно очень обобщенная характеристика. Таких мало. Так же как в других профессиях, хорошо зарабатывают лишь некоторые, в основном люди наделенные лидерскими качествами и целеустремленностью, прожженные в своем бизнесе. Остальные работают для того чтобы платить по счетам своих расходов и среди них немало умных и талантливых людей.

Наука вещь такая, чем больше углубляешься, тем невероятнее и интереснее результат.
Не только наука. В любом другом увлечении то же самое.

luden
12.07.2010, 16:26
Таких мало.
Ну я в общем-то около этой тусовки кручусь. Так вот такие почти все. То есть сверху есть распределители денег, которые вроде как начальники, но это собственно Россия, а вот на нашем уровне маленький филиал рая.
Не только наука. В любом другом увлечении то же самое.
Ну да в принципе. Даже водку пить можно так, что у окружающих глаза на лоб полезут. Но текущие теори действительно впечатляют.

Dragon27
12.07.2010, 19:47
Ну ОТО это наверное перебор
Не так страшен чёрт.

Научпоп интересен именно тем, кто не зубрил ночами ОТО, СТО и прочие науки. Я среди них.
Я не зубрил ночами ОТО, СТО и прочие науки и любил научпоп. Однако позже я (разобравшись, что к чему) сумел в себе выработать интерес к более серьёзной литературе, чем популярной. И этот интерес, благодаря тому, что можно найти в науке, самоподдерживается и самоусиливается.

luden
13.07.2010, 10:30
Не так страшен чёрт.
Я не о том, что она тяжёлая, а что не до конца состыкована.

Dragon27
13.07.2010, 19:33
а что не до конца состыкована
В смысле? Она замечательно состыкована с экспериментом, разве этого мало? А то, что закон сохранения энергии в классическом виде не выполняется или что квантово-полевая модель каноническим образом не строится, это как-то по не особо страшно для обучаемого.

Или ты другое имеешь в виду?

luden
13.07.2010, 22:25
Она замечательно состыкована с экспериментом, разве этого мало?
Я про состыковку с действующими теориями, страшно - не страшно, а диссонанс возникает.

Dragon27
13.07.2010, 22:46
Я про состыковку с действующими теориями
А что же тогда учить, если других нету (проще)? Тяготение Ньютона, которое даже с СТО не стыкуется?
А нестыковка теоретическая, это проблемы теоретические. Они не просто есть, они всегда будут, в любой теории связанной с тем, что ещё неизвестно. Да и несостыковки, в-основном, сильного значения для обычных людей не имеют. Где-то там в крайних приближениях расходимости неренормализуемые.
Это ж не чистая математика.

Да и сама ОТО, разве не действующая? :)

luden
13.07.2010, 23:05
Тяготение Ньютона, которое даже с СТО не стыкуется?
СТО его касается слабее. Опять же, чем в ОТО объясняется факт притяжения объекта в центр искривления?

Dragon27
13.07.2010, 23:16
Опять же, чем в ОТО объясняется факт притяжения объекта в центр искривления?
Как чем? Искривлением ПВ.

luden
13.07.2010, 23:19
Dragon27, да, но искривление само по себе не должно увеличивать вектор движения, оно его искривляет.

Dragon27
13.07.2010, 23:26
да, но искривление само по себе не должно увеличивать вектор движения, оно его искривляет.
То есть? Искривляется не просто пространство, а пространство-время. И 4-мерный вектор, не имеющий пространственных компонент (они нулевые), но имеющий временную, повернётся в ПВ и будет иметь ненулевые пространственные компоненты.

luden
13.07.2010, 23:28
Dragon27, у нас временная компонента преобразуется в пространственную? Что за миндфак?

Dragon27
13.07.2010, 23:35
В СТО при перемене СО временные и пространственные компоненты только так обмениваются метрами. А в ОТО прямой путь по временной оси в кривом ПВ становится путём и по временной части и немного вбок (в пространственные).

luden
13.07.2010, 23:42
Dragon27, я понимаю геометрию искажения четырёхмерного пространства. Просто всегда был уверен, что временные и пространственные координаты вообще не пересекаются. Но если взглянуть с этой стороны, то ОТО представляется мне ещё более фееричным.

Dunkan
13.07.2010, 23:46
В СТО при перемене СО временные и пространственные компоненты только так обмениваются метрами. А в ОТО прямой путь по временной оси в кривом ПВ становится путём и по временной части и немного вбок (в пространственные).
Ох, ну и заумь, тут я бессилен что-либо вставить адекватное и по существу. Создали тему для своих терок понимаешь :\

Dragon27
13.07.2010, 23:49
luden
Временные и пространственные компоненты в ТО (начиная с СТО) - это практически единое целое, и меряются они едиными единицами (задать только масштаб). Эдакое единое 4-мерное пространство-время, в котором все объекты летят вперёд. Переход в другую СО, по сути, это "наклон" точки зрения, при котором временная компонента может частично отойти в пространственную часть. "Наклонить" вертикаль полностью в горизонталь, или хотя бы сильнее, чем линия света, конечно, не получится, и причина этому - псевдоевклидова структура ПВ.
Не знал что ли?

ОТО отличается тем, что эта ПВ в нём имеет "кривую" метрику (тогда как в СТО оно совершенно плоское).

Добавлено через 6 минут
Вот тред (http://e-science.ru/forum/index.php?showtopic=22212&st=0), если кому интересно.

А вот совершенно очевидная картинка:
http://img155.imageshack.us/img155/1937/penrosenewtongravity.png
Роджер Пенроуз "Путь к реальности", читайте! :)

luden
14.07.2010, 10:01
Dragon27, ок, вкурил.

Zosia65
14.07.2010, 10:18
Замечательные теории. Рядом с гравитирующей массой наблюдателю все понятно. А если он в центре оной? Векторы сил тяготения взаимокомпенсированы... При движении к центру, после прохождения горизонта событий, его пространство начинает распрямляться (ну простим ему удар о поверхность). Как вообще материальное тело способно пройти горизонт событий, в котором относительно внешнего наблюдателя вся материя (даже ЭМ излучение!), имеют скорость света... Скорость света, не зависит от скоростей приемника, или источника... Почему же он на выходе не может преодолеть этот барьер? Все таки от чего то зависит?

Сырые они, эти теории. И явно неполные. Истина присутствует, только пока в виде зерен.

luden
14.07.2010, 10:31
Почему же он на выходе не может преодолеть этот барьер?
Искажение пространства загибает вектор направления внутрь.

Zosia65
14.07.2010, 11:17
luden, скорость света постоянна и без относительна. От тела, никогда не способного достичь скорости света, этот самый свет всегда дойдет. В теории ЧД, имеем тело и имеем невозможность прихода от него излучения. Тело еще не достигло, свет уже не способен... Исключение из теории? Теория не полна!

luden
14.07.2010, 12:07
От тела, никогда не способного достичь скорости света, этот самый свет всегда дойдет.
Поставь любую преграду и уже не дойдёт.

Dragon27
14.07.2010, 19:43
А если он в центре оной? Векторы сил тяготения взаимокомпенсированы...
В обычных объектах - да. Если внешняя поверхность Земли под давлением внутренних пород сможет сдерживаться и не проваливаться в центр ЧД, то она сохранит свою форму. В ЧД ничто (давление материи изнутри) не может удержать её тяготение, и решение мат. уравнений показывают, что за очень короткий срок вся звезда быстренько сожмётся в точку в центре ЧД.

При движении к центру, после прохождения горизонта событий, его пространство начинает распрямляться (ну простим ему удар о поверхность).
Какая поверхность? Поверхность уже давно ушла под горизонт (для внутреннего наблюдателя).

Как вообще материальное тело способно пройти горизонт событий, в котором относительно внешнего наблюдателя вся материя (даже ЭМ излучение!), имеют скорость света...
Относительно внешнего наблюдателя падение будет вечно. Относительно падающего, он не сможет обогнать летящий с ним свет (локальную скорость света) и совершенно свободно пролетит ГС.

Почему же он на выходе не может преодолеть этот барьер? Все таки от чего то зависит?
Метрика ПВ не даёт.

Сырые они, эти теории. И явно неполные.
Сырое ваше понимание, и не полное.

Добавлено через 1 минуту
В теории ЧД, имеем тело и имеем невозможность прихода от него излучения.
Если тело под горизонтом, то от него ничего не дойдёт. Если только падает - будет доходить вечно, всё медленнее и тусклее.

Zosia65
15.07.2010, 09:50
В плоскости горизонта событий пространство максимально искривлено. При дальнейшем движении, искривление уменьшается, и становится равным искривлению пространства внешнего наблюдателя. Иначе, в центре гравитационные искривления взаимно компенсируются в любую сторону. В этой точке, пространство растягивается гравитационными силами во все стороны одинаково. Теперь, при отсутствии искривления относительно пространства внешнего наблюдателя, оно имеет ДРУГУЮ РАЗМЕРНОСТЬ (загоните объекты со ВСЕХ СТОРОН, учтите релятивистские изменения длины и времени со ВСЕХ СТОРОН!) Для тела, размеры которого стремятся к нулю, размеры ЧД, становятся сравнимы со вселенной.

В момент наступления факта ЧД, любой материальный объект в центре, будет разорван гравитационными силами до минимально возможного состояния. Все остальное продолжит вечное падение (время стремится остановиться), одновременно принимая исчезающе малые размеры. Ну и что, что до падения между объектами было километровое расстояние, а далее стало миллиметр. Их протяженность тоже уменьшилась в миллион раз. И свет между ними пробежит в миллион раз медленнее (частота не изменится, все таки они достаточно рядом)... И усе. Они в своей вселенной.

Дракоша, прошу не выдергивать отдельные слова, для якобы опровержения их без контекстного смысла. Можно хоть раз услышать Ваше мнение по общему принципу?
В случае его отсутствия, не тратьте время на обличение автора. Можно быть наполненным информацией, находить разночтения с признанными мэтрами... Только это вовсе не означает понимание вопроса.

Dragon27
15.07.2010, 20:14
Иначе, в центре гравитационные искривления взаимно компенсируются в любую сторону. В этой точке, пространство растягивается гравитационными силами во все стороны одинаково.
Это в центре обычного объекта. Но не в центре ЧД. Причину объяснял уже вроде бы 2 раза.

Дракоша, прошу не выдергивать отдельные слова, для якобы опровержения их без контекстного смысла. Можно хоть раз услышать Ваше мнение по общему принципу?
Дело в том, что опровергать всё целиком невозможно, так как это всё представляет собой слабоструктурированный, плохо читаемый бред, уж извините (другие размерности и прочее). Я не могу залезть вам в голову телепатическими связями и выучить вашу собственную физику.
Вы скажите нормально, без излишних извращений - в чём состоит этот ваш загадочный принцип?

Zosia65
20.07.2010, 12:44
Dragon27, ну вот. Нашу расширяющуюся Вселенную, можно рассматривать с позиций наблюдателя падающего в ЧД. То, что осталось за горизонтом (или безнадежно обогнало его в своем падении), и есть загадочные черная материя + энергия. Его размеры непрерывно съеживаются, создавая иллюзию бесконечного расширения конечного(!) объема ЧД. Вот Вам местонахождение и механизм гипотетических параллельных миров... Пространство пластично не только на изгиб, но и на расширение-сжатие. Смотрим на ЧД. Слева, пространство искривлено в одну сторону, справа в другую. А каково оно МЕЖДУ НИМИ?
Математику не сую. Ее и так хватает на сегодня. В частностях. Замахнуться на глобальное описание, будет слишком самонадеянно.

Dragon27
20.07.2010, 18:32
Нашу расширяющуюся Вселенную, можно рассматривать с позиций наблюдателя падающего в ЧД.
Можно, но пока бессмысленно.

То, что осталось за горизонтом (или безнадежно обогнало его в своем падении), и есть загадочные черная материя + энергия.
А вот это уже неверно.
И смешивать их в одно явление не стоит.

Слева, пространство искривлено в одну сторону, справа в другую.
Слева и справа - это где? И почему в разные стороны, обосновать сможете?

Zosia65
21.07.2010, 11:01
Почему нельзя считать материю отстающую от падающего тела со скоростью более скорости света, черной по отношению к нему. В чем я не прав? Гравитация как минимум быстрее света, ЧД массу не теряет во всех теориях.

Слева и справа от центра ЧД, в проекции на плоскость наблюдателя, пространство искривлено в разные стороны. Другие проекции рассматривать?

Рассматривать этот случай бессмысленно... С целью познания?

Dragon27
21.07.2010, 19:13
Почему нельзя считать материю отстающую от падающего тела со скоростью более скорости света, черной по отношению к нему. В чем я не прав?
В том, что это не имеет отношения к той черной материи, которой занимаются астрофизики.

Гравитация как минимум быстрее света
Смотря что вы подразумеваете под скоростью гравитации.

ЧД массу не теряет во всех теориях.
Кроме квантовых.

Слева и справа от центра ЧД, в проекции на плоскость наблюдателя, пространство искривлено в разные стороны.
То есть, одно устремлено влево (к центру), другое вправо (опять-таки к центру)? И что из этого следует?

Zosia65
22.07.2010, 14:33
Следует то, что пространство растягивают: свойства пространства между лево и право, оказываются такими, БУДТО БЫ его растянули. Не знаю чем заменить слово будто бы. Для внешнего наблюдателя, это сфера вполне конечных размеров. Для внутреннего наблюдателя, разве что не бесконечная...

Dragon27
22.07.2010, 20:25
Следует то, что пространство растягивают: свойства пространства между лево и право, оказываются такими, БУДТО БЫ его растянули.
Стянули к центру, растянули с боков. И что?
http://jila.colorado.edu/~ajsh/insidebh/schw_waterfall_s.gif

Zosia65
23.07.2010, 15:02
Длина стрелочек в картинке должна стремиться к нулю при приближении к центру. Вот тогда будет действительно наглядно. Размеры тела по мере приближения к центру, становятся исчезающе малы, то есть пространство вокруг него, растет относительно его размера.
Кстати, а есть данные об уменьшении гравитационных взаимодействий между телами в условиях ускоренного роста расстояния между ними?

Dragon27
23.07.2010, 19:03
Длина стрелочек в картинке должна стремиться к нулю при приближении к центру.
Скорее к бесконечности. А к нулю толщина :)

Размеры тела по мере приближения к центру, становятся исчезающе малы
Я могу точно сказать, что тело будет испытывать растяжение по радиусу за счёт неоднородности гравполя. И что его пекулярная нулевая скорость никакого влияния на его размеры не окажет. Но в самой сингулярности все тела, конечно, сожмутся в точку (условно говоря) благодаря бешеному тяготению :)

Кстати, а есть данные об уменьшении гравитационных взаимодействий между телами в условиях ускоренного роста расстояния между ними?
В условиях любого роста расстояний между телами гравитационное взаимодействие между ними ослабляется.

Zosia65
24.07.2010, 15:50
Релятивистское УМЕНЬШЕНИЕ длины... Растягивают? Значит порвут, УМЕНЬШЕННУЮ!

Наблюдается ли в нашей Вселенной уменьшение гравипотенциалов между материей? Она же расширяется с ускорением. Должна быть четко выраженная тенденция к разбуханию галактик по мере роста расстояний. А некоторые постоянные, - не постоянные, а функции от роста. Не слышал... Впрочем и не считаю себя все познавшем.
Изменение расстояния, в условиях адекватного изменения хода времени, оставит все на своих местах. Физический мир, - это МАТЕРИЯ, ПРОСТРАНСТВО, ВРЕМЯ. По отдельности, эти составляющие не имеют практического смысла. Математика позволяет описать ЛЮБОЙ(!!!) мир. Дело фантазии. Какое отношение сие имеет к реальности?

Dragon27
24.07.2010, 16:57
Наблюдается ли в нашей Вселенной уменьшение гравипотенциалов между материей? Она же расширяется с ускорением. Должна быть четко выраженная тенденция к разбуханию галактик по мере роста расстояний.
Ничего подобного не должно быть. Галактика - слишком сильно гравитационно-связанная система, чтобы расширение могло её разорвать. По крайней мере, в нынешнюю эпоху.

Zosia65
25.07.2010, 14:58
Dragon27. Я говорю об удаленных галактиках (там явление должно быть выражено максимально). И не надо их разрывать. Можно просто рассмотреть строение. Во первых, они должны быть более разряженными. Во вторых, минимальные расстояния между составляющими их телами, изначально должны иметь более жесткие ограничения в сравнении с ближайшими к нам. Иначе они неизбежно объединились бы на ранних этапах.
Итак, удаленные галактики обязаны быть более разряженными изначально. А там что, другие физические законы? Или может законы те же, и они просто имеют более легкие ядра? На периферии, должно так же преобладать наличие тел не входящих ни в какие системы (они взаимодействуют без образования систем, системы уже развалились). Наши методы не позволяют это регистрировать? Это не наблюдаемо пока, по причине огромной временной разницы? Ну тогда я использовал не удачный пример, а факт красного смещения вовсе не обязательно следствие расширения. Возможно он не корректно интерпретирован на основании повседневного опыта, который не может быть применен в системе взаимоисключающих парадоксов. С окраин Вселенной, наша галактика РАСШИРЯЕТСЯ! Ихняя неизменна для них... Безотносительно, никакого расширения нет, относительно - иллюзия оного.

Dragon27
25.07.2010, 15:18
Я говорю об удаленных галактиках (там явление должно быть выражено максимально).
Эта удалённая галактика улетает от нас быстрее - вот это выражение явления. Но сама она не разлетается в стороны быстрее. Расширение на малых расстояниях очень слабое. То есть, сама для себя она такая же, как и мы для себя. С чего они должны быть более разряжёнными? Разве только по той причине, что многие из них мы видим на более ранней стадии образования. Но уже где-то порядка 1-2 миллиарда лет после условного события БВ были образованы первые галактики.

На периферии, должно так же преобладать наличие тел не входящих ни в какие системы (они взаимодействуют без образования систем, системы уже развалились).
То есть? Какие системы уже развалились? Если вы про периферию видимой Вселенной, может логичнее будет говорить про ещё не развалились?
Квазары расположены на разных расстояний, но больше всего их на периферии :)

факт красного смещения вовсе не обязательно следствие расширения. Возможно он не корректно интерпретирован на основании повседневного опыта, который не может быть применен в системе взаимоисключающих парадоксов.
Ну он интерпретировался не только на основании повседневного опыта, а прочие интерпретации (усталый свет, например) попросту не прошли отбора, либо имеют гораздо меньше наблюдательных подтверждений.

Zosia65
25.07.2010, 19:46
Вернемся к теме раздела. Возможен ли сверхсветовой переход. ЧД показывают, такое возможно. Раз свет не может долететь от тела из под горизонта событий, значит тело там имеет скорость... :))
Чего там, бесконечный рост массы? А нету релятивистской массы. Без относительность скорости света? А красный мы раньше перестанем фиксировать, чем синий. В какой теории найдем опровержение? Ньютоновская не верна на таких скоростях, ОТО не верна на квантовых уровнях... Во всяком случае пока можно. Единая теория поля расставит все по своим местам.

Dragon27
25.07.2010, 19:53
Возможен ли сверхсветовой переход. ЧД показывают, такое возможно.
Сверхсветовое движение в ОТО не эквивалентно сверхсветовому движению в СТО.
http://forum.igromania.ru/showpost.php?p=8100216&postcount=2

ОТО не верна на квантовых уровнях..
Зато работает на горизонте событий.

Zosia65
26.07.2010, 00:06
Dragon27, посчитал возможным не как следствие какой то теории. Сам факт существования ЧД, пропадание в них тел вместе с излучением, говорит о такой возможности. Разница то... Значит то же возможно простым разгоном (ускоряющем воздействии) тела. И вот, нету его у нас. И света от него нет. Одна масса осталась. Черная :)) . Если это наш корабль, и он там найдет чем заправиться, то может вынырнуть в вполне предсказуемом месте.

Много говорят о множестве возможных параллельных миров. А вот где они, не понятно... И масса черная имеется... И пример с ЧД... Остался шаг. Ускорение наблюдателя не менее скорости света за секунду (ведь тело продолжает существовать за горизонтом событий). И черной станет наша Вселенная, а бывшая черной до этого, станет светимой. Продолжая свой путь тем же способом, он вернется в начальную точку.

Dragon27
26.07.2010, 00:51
Значит то же возможно простым разгоном (ускоряющем воздействии) тела.
Что возможно простым разгоном? Уж не сверхсветовое ли движение?

Много говорят о множестве возможных параллельных миров. А вот где они, не понятно...
Ну так говорят много чего. Вот (http://en.wikipedia.org/wiki/Multiverse#Tegmark.27s_classification), например.

Zosia65
26.07.2010, 03:31
Ускорение= скорость света за секунду. Длительность= менее секунды.. Есть разница между телом под действием гравитационного и обычного ускорения?
Dragon27 "Много говорят" - это повод еще поговорЯть? Для меня сие было поводом обосновать это "Много говорят". Ну не знаю я английского, остаюсь в неведении относительно Вашего примера. Впрочем уже говорил Вам об этом, и о бессмысленности попыток использовать машинные переводы.

Dragon27
26.07.2010, 19:10
Есть разница между телом под действием гравитационного и обычного ускорения?
В ОТО есть. И довольно серьёзная. Одно дело - толкать тело силой. И совсем другое - тело само летит по кривой геодезической. Как по прямой по инерции, так и по кривой - по инерции.

Ну не знаю я английского, остаюсь в неведении относительно Вашего примера.
Там вроде и машинный перевод можно заюзать (всё равно толком не распишут, это ж вики, да ещё и тема не особо благодатная).

Zosia65
28.07.2010, 17:11
Разница появляется когда мы переходим от рассмотрения материальных точек, к реально пространственно заполняющим телам. Разница в характере приобретения результатирующей центром масс. То есть разница переходных состояний в объекте, но никак не самого объекта для внешнего наблюдателя. Тело ускоряемое до скорости света за секунду (оно состоит из заряженных частиц и ускоряется электрическим полем, от тела много большей массы и имеющего околосветовую скорость, ну на всяк :)) ). Оно находится в состоянии аналогичном пребыванию в плоскости ГС? Ну причем здесь деформации, коль массу все же вынуждают. Это то по разному отражается на его локальном пространстве времени?

Dragon27
28.07.2010, 20:12
Тело ускоряемое до скорости света за секунду (оно состоит из заряженных частиц и ускоряется электрическим полем
Нет, до скорости света вы тело не ускорите.

Разница появляется когда мы переходим от рассмотрения материальных точек, к реально пространственно заполняющим телам. Разница в характере приобретения результатирующей центром масс. То есть разница переходных состояний в объекте, но никак не самого объекта для внешнего наблюдателя.
А вы можете этот абзац нормальной речью объяснить? Что там такого мы приобретаем, когда переходим от мат. точек к реальным телам в нашем случае?

Zosia65
31.07.2010, 04:35
Извиняюсь, не до скорости света, а с ускорением равным скорость света за секунду.

Далее перевожу нормальной речью: находясь внутри тела, мы не сможем различить толкают его с ускорением, или притягивают.

Dragon27
31.07.2010, 18:35
Извиняюсь, не до скорости света, а с ускорением равным скорость света за секунду.
Локальным. И что с ним, этим телом, будет? Наверняка его очень жёстко будет плющить (из-за ускорения, а не скорости), если оно не равномерное, не как в однородном гравполе, например.

находясь внутри тела, мы не сможем различить толкают его с ускорением, или притягивают.
В локальном случае, да. В реальном мире - трудно, но возможно :)

Zosia65
02.08.2010, 01:27
Dragon27 Сидят два крокодила на дереве. Один умный, а второй устал. Вдруг вжжж. Напильник пролетел. Сидят дальше, им то что. Тут фррр. Стая напильников подлетела.
- Эй, крокодилы. Здесь напильник не пролетал?
- Ну пролетал...
- А куда он полетел?
- На юг...
Фрруу. Улетели. Усталый не удержался, спрашивает:
- А чего ты им наврал? Он же на север полетел.
- Да какая разница, все равно не догонят...
- Почему?
- Так он без ручки был...
Плющить будет для внешнего наблюдателя. Тело в расплющенном пространстве, своей сплюснутости... Ну никак.
В ответ, что угодно. Лишь бы не по существу. Политик пропадает.

Dragon27
02.08.2010, 01:48
Плющить будет для внешнего наблюдателя.
Я не про Лоренцево сокращение (которое относительно), а про обычное сжатие тела (перегрузки) под воздействием обычного ускорения. Оно будет совершенно объективно.

Zosia65
02.08.2010, 02:03
Dragon27, да объективно. Так же объективно разрушение тела под действием собственного веса на поверхность достаточно массивного объекта. С пространством как. С относительным при ускорении. Может тело проявлять свойства ЧД к наблюдателю вдоль вектора к нему?

Dragon27
02.08.2010, 12:25
Может тело проявлять свойства ЧД к наблюдателю вдоль вектора к нему?

Если тело ЧД, то оно будет проявлять свойства ЧД. Если тело не ЧД, то оно не будет ЧД ни в какой системе отсчёта (по крайней мере не припомню таких хитрых экстремальных случаев, ни в одной другой ИСО точно).
А вот если тело ускоряется, то в его системе отсчёта можно заметить, например, горизонт событий, вызванный ускоренным движением.

Zosia65
05.08.2010, 00:46
Dragon27, вот на появление ГС в одном измерении и хочу обратить внимание. Для части материи из мира наблюдателя, станет невозможным электромагнитный обмен с рассматриваемым телом (на примере ЧД знаем, гравитационные взаимодействия не изменятся). Вероятнее всего проявится некоторая часть материи, которая имела до этого черные свойства.

Dragon27
05.08.2010, 00:55
вот на появление ГС в одном измерении и хочу обратить внимание.
(в одном измерении?) Обратим. Задайте ситуацию (тело ускоряется? тело падает в ЧД? что вы имеете в виду), и что вы хотите в ней выяснить.

Zosia65
18.08.2010, 04:36
Dragon27, тело улетает от нас с околосветовой скоростью. Вопрос: наблюдатель с него увидит Вселенную идентичную нашей? Увидит ли он часть материи с черными свойствами для нас (тупо сравнявшись с ней по скорости)?

Dragon27
18.08.2010, 15:37
тело улетает от нас с околосветовой скоростью
С постоянной?

Вопрос: наблюдатель с него увидит Вселенную идентичную нашей?
Конечно. За исключением резкого сокращения расстояний вдоль его полёта, сильной анизотропии реликтового излучения и других подобных банальностей. темную материю он также не увидит, как и мы.

Если же вы имеете в виду ускоряющегося наблюдателя, то вместо вакуума он будет видеть замечательное тепловое излучение.

Zosia65
19.08.2010, 13:54
Доплеровский эффект, имеет крайним значением бесконечную длину ЭМ волны. Понятно, что материя при крайних скоростях будет выглядеть черной. Что запрещает ее догнать по скорости, оставив начальный мир с черными свойствами? Есть конкретные ограничения?

Dragon27
20.08.2010, 01:18
Доплеровский эффект, имеет крайним значением бесконечную длину ЭМ волны.
Это в каком случае?

Zosia65
20.08.2010, 10:01
При разлете со скоростью света. Странный вопрос... А Вы думаете? Или механически ассоциируете...

Dragon27
20.08.2010, 18:15
То есть, в случае обычного движения (не в результате расширения пространства)? Правда, при обычном движении скорость света недостижима :)
Тогда "начальный мир" будет для нас выглядеть, как я уже сказал, очень сильно анизотропно (вы ведь знаете, что CMBR имеет анизотропию из-за движения нашей планеты?). Часть неба будет очень яркой, горячей, часть совершенно тусклой и холодной.
Перед глазами стоит картинка дипольной анизотропии, но, к сожалению, из-за плохого качества интернета на моём ноутбуке, я её пока не могу найти и вставить (посты с трудом отгружаются).

Zosia65
21.08.2010, 01:53
При обычном движении тела в расширяющемся пространстве. Крайние скорости - около световые. Наша планета имеет такую только по отношению к граничной материи Вселенной. При росте скорости наблюдателя, все больше граничной материи по ходу движения он увидит (без предельного для восприятия Доплеровского смещения), соответственное количество выпадет из наблюдения сзади. Если он наберет около световую скорость по отношению к ближайшим соседям, то они просто станут границей его видимой Вселенной.

Dragon27
21.08.2010, 01:57
Наша планета имеет такую только по отношению к граничной материи Вселенной.
Что ещё за граничная материя Вселенной?

Zosia65
21.08.2010, 02:51
Чем дальше от нас, тем выше скорость разлета. На границе Вселенной она приближается к световой. Потому так и обозвал, извиняюсь общепринятые определения не все знаю.

Dragon27
21.08.2010, 03:24
Чем дальше от нас, тем выше скорость разлета. На границе Вселенной она приближается к световой.
Раньше. А объекты, находящиеся на границе видимой части Вселенной, нагло превышают скорость света. И мы их видим.

Zosia65
21.08.2010, 13:16
Dragon27, ой, превышение скорости света... А какие релятивистские эффекты? Специально не лезу в эту заобласть... Не вру, о появлении мнимого пространства в мнимом времени заикался. Время, как и пространство, зависит от гравитационной напряженности. Масса ЧД не меняется со временем (где релятивистский рост массы?). Получается лишь рост кинетической энергии, или массы в одном измерении,- вдоль вектора движения. Но это не трехмерная масса. Формулы Энштейна, описывают только одно измерение! Рассматривая в своей ИСО улетающий объект, приходится уменьшать его пространство в останавливающемся времени, для сохранения постоянства скорости света и у него. Начальный постулатик изменить бы. Не зависимость массы от ИСО. Не материальной точки. Трехмерного объекта. Тогда разлет, станет частным случаем. Тогда понятным станет зависимость скорости света от среды, и не зависимость массы от оной. Да многое станет на свои места наглядным образом. Я не оспариваю ТО, считаю неудачной отправную точку. Можно путь пешехода выразить количеством колебаний звуковой волны в воздушной среде. Целесообразно ли?

Dragon27
21.08.2010, 18:36
Dragon27, ой, превышение скорости света... А какие релятивистские эффекты? Специально не лезу в эту заобласть... Не вру, о появлении мнимого пространства в мнимом времени заикался.
А лучше и не заикайтесь, они тут вообще ни при чём. Пекулярная скорость у объектов стандартная, до 1000 км/с, примерно.

где релятивистский рост массы?
Забудьте вы вообще это словосочетание. Нет в современной физике релятивистской массы. Лишняя она, только народ недоученный путает.

Формулы Энштейна, описывают только одно измерение!
Рассмотрение движения в одном измерении просто удобнее. А формулы спокойно применяются для любого нужного числа измерений.

Рассматривая в своей ИСО улетающий объект, приходится уменьшать его пространство в останавливающемся времени, для сохранения постоянства скорости света и у него. Начальный постулатик изменить бы.
Не получится, он уже надёжно подтверждён, и никуда исчезать не собирается.

Тогда понятным станет зависимость скорости света от среды, и не зависимость массы от оной.
Они и так совершенно ясны и понятны.

Zosia65
23.08.2010, 11:05
Dragon27, мне не понятно :))
Надежно подтвержден,- да и не спорю. Надежно висит в ступоре, хоть и наглядна кому то. Для истины, нет ничего невозможного, можно рассчитать параметры орбиты земли вокруг солнца, элементарно изучая характеристики прыжков блохи, на любимой собаке. Если собака не будет стоять на месте (прочь ИСО ее и блохи), мы узнаем о шарообразности земли. Как грится, пути познания неисповедимы...

Dragon27
23.08.2010, 16:48
мне не понятно
Фотон летит, "врезается" в атомы среды (точнее, поглощается и переизлучается), поэтому общее расстояние преодолевается медленнее. Так понятнее? Это если очень популярно (на самом деле всё, конечно, гораздо сложнее, включая не только объяснения, но и различные виды скоростей: фазовая, групповая). Почитайте Фейнмана, он там описывает рассуждения, с помощью которых теоретически выводит показатель преломления (примерную формулу) для среды.

Надежно подтвержден,- да и не спорю. Надежно висит в ступоре, хоть и наглядна кому то.
Для наглядности стоило бы хотя бы основательно разобраться в вопросе.

Zosia65
28.08.2010, 11:10
Переход энергии фотона в материальное тело (поглощение), и НАОБОРОТ, как раз и указывает на возможность трансляции массы излучением. На это же указывает феномен дефекта масс, - прямой переход части массы материального тела в энергию излучения. По теме форума, получаем,- трансляция массы возможна, ограничения связаны постулатом скорости света. Для ретрансляции, нун сначала совершить обратный переход, перевод излучения в массу. Думается вполне возможно, просто пока технически не доступно.

Dragon27
28.08.2010, 13:45
Переход энергии фотона в материальное тело (поглощение)
Вы так говорите, как будто фотон не материален.

По теме форума, получаем,- трансляция массы возможна, ограничения связаны постулатом скорости света. Для ретрансляции, нун сначала совершить обратный переход, перевод излучения в массу. Думается вполне возможно, просто пока технически не доступно.
И каким образом вы превратите сложное комплексное тело в излучение, а потом в обратно в точно такое же тело? Только так (http://ru.wikipedia.org/wiki/%D0%9A%D0%B2%D0%B0%D0%BD%D1%82%D0%BE%D0%B2%D0%B0%D 1%8F_%D1%82%D0%B5%D0%BB%D0%B5%D0%BF%D0%BE%D1%80%D1 %82%D0%B0%D1%86%D0%B8%D1%8F) (но тут, всё-таки, по-другому, да ещё запутанные пары нужно заранее создавать, так что не выйдет). Больше атома ничего толком передать не можем.

А тема форума сводилась к перемещениям со сверхсветовой скоростью.

Zosia65
29.08.2010, 10:50
Не правильно сказанул. Имел ввиду переход материи излучения в материю с массой покоя. О практическом исполнении тож не говорил. Просто теоретическая возможность существует. Ограничения по скорости света, возможно обходятся изменением частоты передачи. Для приемника осуществляющего обратный переход, это должно иметь значение. Красное смещение, или синее до рентгеновского. Одного и того же объекта. Без разрушения, переводом массы покоя в энергию излучения.

Архитектор
29.08.2010, 15:09
...превысить скорость света если и удастся, то что получится после остановки? приблизится к скорости света теоретически можно используя нейтронный двигатель-в котором будут сталкиватся антитела с материей, опять же при слиянии даже одного нейтрино с антинейтрино на околоземной орбите приведет к сожжению земли..

Dragon27
29.08.2010, 15:42
Ограничения по скорости света, возможно обходятся изменением частоты передачи.
Что?

Без разрушения, переводом массы покоя в энергию излучения.
И как тут не разрушить структуру?

приблизится к скорости света теоретически можно используя нейтронный двигатель-в котором будут сталкиватся антитела с материей
Что это такое, и где вы про это прочитали?

при слиянии даже одного нейтрино с антинейтрино на околоземной орбите приведет к сожжению земли..
Ну это вы перегнули.

Zosia65
30.08.2010, 05:17
Dragon27. Это массу Вы не разгоните выше "с". А вот разница частоты принятого приемником сигнала, характеризует его скорость относительно передатчика. Короче, масса то переведена в излучение. Совершите обратный переход на другой частоте, и получите то же тело но с другой скоростью. Ну включите мозаГ. Требуется два преобразования. С частотой ограничений нет.

Архитектор, остановка ничем не отличается от разгона. Впрочем возможно я не понял вопрос.

Dragon27
30.08.2010, 12:18
А вот разница частоты принятого приемником сигнала, характеризует его скорость относительно передатчика. Короче, масса то переведена в излучение. Совершите обратный переход на другой частоте, и получите то же тело но с другой скоростью. Ну включите мозаГ. Требуется два преобразования. С частотой ограничений нет.
Ну не получается у меня включить мозаГ. Каким образом мы преодолеем световой барьер (пекулярной скоростью)? Массивная материя - не быстрее света. Безмассовая - ровно скорость света.

Zosia65
03.09.2010, 05:30
Dragon27. Ой как все строго :)) Не превысим, каюсь, вредничал. Превышение скорости света, означает выход за пределы причинности настоящего. Ничего не может уйти из него ни в прошлое ни в будущее не нарушив законы сохранения. Мы не можем нулевую частоту (удаление со скоростью С) уменьшить еще, некуда. Увеличение, так же должно иметь граничный предел (Доплеровский эффект при сближении), и по той же причине. Насчет массовости безмассовости, хочу обратить внимание на факт наличия черной материи. ЭМ взаимодействия нет. Гравитационное пожалуйста. Масса со скоростью света? Факт. Кстати необъяснимый с точки зрения богоподобной ОТО.

Dragon27
03.09.2010, 18:38
Мы не можем нулевую частоту (удаление со скоростью С) уменьшить еще, некуда.
А что это у нас летит со скоростью света и имеет нулевую частоту?

Насчет массовости безмассовости, хочу обратить внимание на факт наличия черной материи. ЭМ взаимодействия нет. Гравитационное пожалуйста. Масса со скоростью света? Факт.
С чего это? Dark Matter у нас со скоростью света не летает.

Zosia65
04.09.2010, 07:59
От нас удаляется объект, испускающий в нашу сторону свет. Это по вопросу о нулевой частоте ЭМ...

"С чего это? Dark Matter у нас со скоростью света не летает", если утверждение не голословно, то пора на Нобелевскую очередь занимать. Может я что пропустил, и ученному миру уже известно нечто большее, чем наличие какой то там темной материи... Кстати, окромя озвученной мною версии (материя со скоростью света), есть и еще одна,- микро ЧД. Кто знает, может на квантовом уровне они друг дружку отталкивают. Кирпичику из микро ЧД, легко будет затеряться в необъятных просторах, вот и не могут найти... 80% :))

Dragon27
04.09.2010, 18:40
От нас удаляется объект, испускающий в нашу сторону свет.
Но со скорость света он от нас удаляться не может. Иначе он либо света, либо какая-либо другая безмассовая НЕХ.

"С чего это? Dark Matter у нас со скоростью света не летает", если утверждение не голословно, то пора на Нобелевскую очередь занимать.
Голословно противоположное утверждение. Энергии безмассовой материи просто не хватит для подобных гравитационных фокусов.
Даже скромная википедия с вами не согласна:
В зависимости от скорости частиц различают горячую и холодную тёмную материю. Горячая тёмная материя состоит из частиц, движущихся с околосветовыми скоростями, по-видимому, из нейтрино.

Горячей тёмной материи недостаточно, по современным представлениям, для формирования галактик. Исследование структуры реликтового излучения показало, что существовали очень мелкие флуктуации плотности вещества. Быстро движущаяся горячая тёмная материя не могла бы сформировать такую тонкую структуру.

Холодная тёмная материя должна состоять из массивных медленно движущихся (и в этом смысле «холодных») частиц или сгустков вещества. Экспериментально такие частицы не обнаружены.

В качестве кандидатов на роль холодной тёмной материи выступают слабо взаимодействующие массивные частицы (Weakly Interactive Massive Particles, WIMP), такие как аксионы и суперсимметричные партнёры-фермионы лёгких бозонов — фотино, гравитино и др.
Лучше конечно вам английскую вики почитать:
Composition of Dark Matter (http://en.wikipedia.org/wiki/Dark_matter#Composition)

Zosia65
09.09.2010, 13:33
Голословны оба утверждения. Dark Matter, пока область догадок и предположений. Думается ограниченность ОТО (или трактовка выводов из нее), тому причина. Почему, темная материя не входит в состав обычной материи? ПОЧЕМУ ОНИ НЕ ПЕРЕМЕШАНЫ? Ведь гравитационное взаимодействие между ними происходит от рождения Вселенной. Невозможность иных взаимодействий, все равно не мешает их пространственному слиянию, более того, оно должно было быть неизбежным. Остается один выход,- темная материя не имеет массы покоя. Есть другие предположения?

Dragon27
09.09.2010, 18:37
Голословны оба утверждения.
Учёные, в отличии от форумных мыслителей, не только умело рассуждают, но и ещё умело наблюдают.

Почему, темная материя не входит в состав обычной материи? ПОЧЕМУ ОНИ НЕ ПЕРЕМЕШАНЫ? Ведь гравитационное взаимодействие между ними происходит от рождения Вселенной.
С чего они должны быть перемешаны? Через вас регулярно пролетают несметные количества нейтрино, вы их замечаете? А ведь они взаимодействуют слабым взаимодействием (и гравитационным, конечно же). А вот тёмная материя, судя по всему, взаимодействует только гравитационным (которое слабее слабого в очень много раз). Однако их гравитационное влияние привело к тому, что видимая материя кучкуется в местах скопления dark matter.

Простите, что без цифр.

Zosia65
11.09.2010, 09:50
В образовании планет, звезд гравитация являлась доминирующим фактором. Даже наличие булыжника, ее следствие. Почему dark matter, о которой достоверно известно наличие гравитации, не вошла в состав распространенной материи? ПОЧЕМУ? Есть объяснение другое, кроме отсутствия массы покоя? Вижу только одно, НАШИ ПРЕДСТАВЛЕНИЯ О МАТЕРИИ НЕ ПОЛНЫ. Мягко...

Цифирь не надо. Разве что с однозначными выводами.

Dragon27
11.09.2010, 16:19
В образовании планет, звезд гравитация являлась доминирующим фактором. Даже наличие булыжника, ее следствие. Почему dark matter, о которой достоверно известно наличие гравитации, не вошла в состав распространенной материи?
В образовании крупных структур, может быть, и играла. Но для того, чтобы быть надёжно замеченой не по гравитационному влиянию она должна обязательно участвовать и в других взаимодействиях.
Или мне вам объяснять, почему звёзды светятся?

Есть объяснение другое, кроме отсутствия массы покоя?
С чего вы взяли, что у Dark Matter нет массы покоя?

Zosia65
13.09.2010, 18:08
Dragon27, почему ни в одном веществе, не обнаруживается масса без признаков иных взаимодействий? Даже пресловутый дефект масс, имеет другое объяснение.

По массе покоя Dark Matter, енто вопрос, а не утверждение.

Dragon27
13.09.2010, 19:49
По массе покоя Dark Matter, енто вопрос, а не утверждение.
Dark Matter возможно состоит из частиц с немалой массой покоя, вот моё утверждение.

почему ни в одном веществе, не обнаруживается масса без признаков иных взаимодействий?
Объясните подробнее.